You are on page 1of 88

P hysi cs | 4.

35

Solved Examples

JEE Main/Boards Sol: In the first case, the blocks A and B move together.
The friction force will be exerted on the bottom surface
Example 1: A spherical shell is of B. In the second case only block B moves. The friction
resting against the vertical wall force will be exerted both on bottom and top surface
which makes an angle 30° with of block B.

30°
the vertical as shown in the
Figure. Determine the normal Let us consider free diagrams of A and B as two separate
reaction at the wall and tension systems shown as follows:
in the string. NA NA

Sol: Draw the FBD of the sphere. m=5kg


Resolve the forces in horizontal F NA
B
and vertical directions. Apply (NA+NB)
A
Newton’s first law along the
horizontal and vertical directions. NB
(mA+mB)g mAg
T
T cos 30°

A
30°

F B
T sin 30° N

(a) =
WA m
=Ag 4 N ; WB m
= =Bg 8 N

mg NB =NA + mB g =(mA + mB )g = 4 + 8 = 12N


F=
µ × 12 =
0.25 × 12 =
3N
FBD of the sphere is provided. Since sphere is in
equilibrium, hence, (b) F =µ(NA + NB ) because B is sliding over horizontal
N – T sin30 = 0  … (i) rough surface under it and B will be sliding under A also.
= µ(mA + mB ) + µ mA g =
µ(2mA + mB )g = µ(2 WA + WB )
and ⇒ T cos30 =
mg  ...(ii) ... (ii)
= 0.25(8 + =
8) 4N
mg 5 × 10 × 2  100 

= T = =   N.
cos30 3  3 
Example 3: Two blocks each
100 1 50 having mass of 20 kg rest on
from (i) N= T sin30= × = N.
3 2 3 frictionless surfaces as shown
in the Figure. Assume that the A
Example 2: The Figure below shows blocks A and B pulleys to be light and
weighing 4N and 8N, respectively and the coefficient frictionless. Now, find (a) the B
of sliding friction between any two surfaces is 0.25. Find time required for the block A to
the force necessary to drag the block B to the left with move 1 m down the plane, starting from rest and (b) the
constant velocity in all the cases when (a) A is kept over tension in the cord connecting the blocks.
B and (b) A is held firmly over B. Sol: For block A apply Newton’s second law of motion
along the inclined and for block B apply Newton’s
second law along the horizontal.
A A T
Both the blocks A and N

F B F B B are considered as two


independent systems. The MA
mAgsin
FBDs for the blocks A and B mAgcos
are shown in the Figure and  mAg
4 . 3 6 | Forces and Laws of Motion

Figure where T is tension in the string. = A1 + A2 = 4 + 1 = 5N.sec


mA gsin θ − T = mA a  ......(i) ... (i)
Momentum at t = 3 sec =5N sec (at t = 0, P = 0)
=N mA gcos θ  ...(ii) ... (ii) 5
mv = 5 ; v= = 2.5m / sec.
2
T = mB a  ...(iii) ... (iii)
Now, by adding equations (i) and (iii), we obtain Example 5: Three blocks of masses, and are connected
by inextensible strings passing over three massless
T a pulleys as shown in the Figure. The coefficient of friction
N between the masses and horizontal surfaces is µ.
mBAT Assume that M1 and M2 are sliding. Now, find
B (a) Relation between accelerations a1, a2 and a3

mBg (b) Tension T in the strings

mA gsin=
θ (mA + mB )gsin θ x₁
a₁ a₂
x₂

 mA   20  3 2
=⇒a =
  gsin θ =  (10)   3 m / s x₃ x₃
m
 A + mB   20 + 20   5 
1
1  2s  2  1
(a)s = at2 ; t =   =  2 ×  = 0.82 M₂a₃
2  a  3
a₁ a₂
(b) T = mB a = 20 × 3 = 60N.
N₁ N₂

Example 4: A body of mass = 2 kg starts from rest T T


f M₁ M₂ f
whose force time graph is shown in the following graph. x₁ x₂
A B
(a) What is momentum of the body at t = 4 seconds? M₁g
T
M₂g
T
(b) What is velocity of the body at t = 3 seconds? x₃ x₃

Sol: Area under the force-time graph and the time axis a₃
is equal to the change in momentum. x
M₃ y
F
(N)
M₃g
2N
t =3 t =4 Sol: To find the constraint relation between the
t =2 t (sec) acceleration of the blocks, measure the distances of
blocks from the stationary pulleys. Draw the FBD for
–3N each block. For M1 and M2 apply Newton’s second law
in horizontal direction. For M3 apply Newton’s second
(a) Area under the curve from t = 0→2 sec. law in vertical direction.

A1 = 2 × 2 = 4N.sec. Area from t = 2→3 sec. (a) Forces of friction f, tension T and reaction are marked
for the blocks M1 ,M2 and M3 .
1
A2 = × 1 × 2 = 1N.sec. Now, take the horizontal line AB as the reference line,
2
i.e., x-axis and vertically downward as y-axis.
Area from t = 3→4 sec.
If x1 , x2 and x3 are the lengths of the strings, then
1
A3 =− × 1 × 3 =−1.5N.sec.
2 x1 + x2 + 2x3 =
L where L is the constant length of the
string.
Therefore, the net impulse = 4 + 1 – 1.5 = 3.5 N sec
Now, differentiating twice, a1 + a2 + 2a3 =
0
=Pƒ impulse +=
Pi 3.5 + 0 = 3.5 N.s or kg.m / s
As a3 is increasing, a1 and a2 are decreasing.
(b) Impulse from t = 0→3 sec
P hysi cs | 4.37

Thus, the constraint relation shows that (a) Let L be the length of the string. Let x1 be the length
a1 + a2 =
2a3 of the vertical string and x2 be the length of each string
in the horizontal direction. The constraint relation for the
(b) The equations of motion are given as follows string of length L is x1 + 2x2 = L Now, by differentiating
twice, a1 + 2a2 = 0
For M3 , M3g − T − T =M3a3  ...(i) ... (i)
If a1 is +ve, then a2 is –ve,
For M1 , T − µM1g =M1a1  ....(ii) ... (ii)
a a1
For M2 , T − µM2g =M2a2  ...(iii) a1 − 2a2 =0 or a2 = =
... (iii) 2 2
a
a1 + a2 =
2a3 
...(iv) ... (iv) Let a1 = a be the acceleration of M and be the
2
acceleration of 2M.
2T
Dividing (i) by M3 , g − a3
= ∴ Mg – T = Ma  … (i)
M3
T a
=2T 
2M × ...(ii) ... (ii)
Dividing (ii) by M1 , − µg = a1 2
M1
Ma
=2 T Ma
= or T
T 2
Dividing (iii) by M2 , − µg = a2
M2 Now, by substituting for T in (i)
Using (iv), a1 + a2 =
2a3
Ma 3Ma 2g
Mg − = Ma; = Mg; ∴ a=
T T 4T 2 2 3
− µg + − µg
= 2g −
M1 M2 M3 Ma 2gM Mg
(b)=
T = =
2 2×3 3
 1 1 4 
T + +  = 2µg + 2g
= 2g µ + 1 5Mg
 M1 M2 M3  (c) Force on clamp
= C (2 T)2 + (T)
= 2
5T
= .
3
2g(µ + 1) (µ + 1)g
T= = Example 7: Masses M1 ,M2 and M3 are connected by
1 1 4 1 1 2
+ + + + strings of negligible mass which pass over massless and
M1 M2 M3 2M1 2M2 M3
frictionless pulleys P1 and P2 as shown in the Figure.
The masses move such that the portion of the string
Example 6: Masses M and 2M are connected through between P1 and P2 is parallel to the incline and the
pulleys A and B with strings as shown in the Figure. portion of the string between P2 and M3 is horizontal.
Assume that both the pulleys and the strings are light The masses M2 and M3 are 0.4 kg each and the coefficient
and all the surfaces are frictionless. of kinetic friction between masses and surfaces is 0.25.
(a) Find the acceleration of the block of mass M. The inclined plane makes an angle of 37° with the
horizontal, however, the mass M1 moves with uniform
(b) Find the tension in the string.
velocity downwards. Now, find
(c) Calculate the force exerted on the clamp.
(a) The tension in the horizontal portion of the string
Sol: To find the constraint relation between accelerations
of blocks M and 2M, measure all distances from the (b) The
= mass M1 g 9.8=(
ms−2 ,sin 37° 3 / 5 . )
fixed pulley A. Apply Newton’s second law in horizontal P1
direction for block 2M and Newton’s second law in T1
M2 M
vertical direction for block M. 2 gc
 = 0.25
T1 os

a₂ T2
x₂ Mg P2
M3
2M T2 mMg
B T M1 37°
T A a₁

Clamp T x₁ Sol: Apply Newton’s first law for each of the blocks as
the velocity of each block is constant.
M
Let T1 be the tension between M1 and M2 and T2 be
Mg the tension between M2 and M3 .
4 . 3 8 | Forces and Laws of Motion

Let µ be the coefficient of kinetic friction. Then Example 9: In the adjacent Figure, masses of A, B and
(a) T2 =
µM3g =
0.25 × 4.0 × 9.8 =
9.8N C are 1 kg, 3 kg and 2 kg, respectively. Find (a) the
acceleration of the system and (b) tension in the string.
(b) T1 = M1g 2
Neglect friction ( g = 10m/ s )
T1 − T2 − µM2gcos θ − M2gsin θ = 0

4 3
M1g − 9.8 − 0.25 × 4 × 9.8 × − 4 × 9.8 × =0
5 5
60° 30°
4 12
M1 × 9.8 = 9.8 + 9.8 × + 9.8 × =0
5 5 Sol: Draw the FBD of each block and apply Newton’s
4 12 21 second law along the incline plane for each block.
∴ M =1 + + = = 4.2 kg
1 5 5 5 (a) In this case, the net pulling force

Example 8: A rod AB rests with = mA gsin60° + mB gsin60° − mC gsin60°


Y
the end A on rough horizontal
3 3 1
ground and the end B against = (1)(10) + (3)(10) − (2)(10) = 24.64N
2 2 2
smooth vertical wall. The rod is of
uniform length and of weight W. If Therefore, the total mass being pulled = 1+3+2 = 6 kg
the rod is in equilibrium in the 21.17
30° A ∴ Acceleration of the system
= a = 4.1m / s2
position shown in the Figure, then O X 6
find: (b) For the tension in the string between A and B.
(a) Frictional force at A
T2 a
(b) Normal reaction at A
(c) Normal reaction at B. Y
mCg sin 60°

Sol: For translational equilibrium, B NB FBD of A is mA gsin60° − T1 =


= (mA )(a) mA (gsin60° − a)
the vector sum of all the forces
acting on the rod is zero. Take  3 
T1 (1)  10 ×
∴= − 4.1=
 4.56N
component of forces along NA  2 
W  
horizontal (x-axis) and vertical
(y-axis) direction. Sum of O
30° A
X (b) For the tension in the string between B and C. FBD of
fA
components of forces along the x C, T2 − mc gsin30° =mc a
and y axes will be zero. For rotational equilibrium, the   1 
net torque of all the forces acting on the rod relative to T2 2 3.53 + 10  =
∴ T2= mc (a + gsin30°) ∴=   18.2N
  2 
a fixed point (say O) is zero.
Let the length of the rod be 2l. Using the three conditions
of equilibrium, the anticlockwise moment is taken as Example 10: A small smooth ring of mass m is threaded
positive. on a light inextensible string of length 8L which has its
ends fixed at points in the same vertical line at distance
(i) ∑ Fx = 0 ∴ NB=− fA 0 or
= NB fA  … (i) 4L apart. The ring describes horizontal circles at constant
(ii) ∑ Fy =∴ 0 NA − W =0 or NA =W  … (ii) speed with both parts of the string taut and with the
lower portion of the string horizontal. Find the speed
(iii) ∑ τ0 = 0
of the ring and tension in the string. The ring is then
∴NA (2lcos30°) − NB (2lsin30°) − W(lcos30°) =0 tied at the midpoint of the string and made to perform
horizontal circles at constant speed of 3 gL . Find the
3 tension in each part of the string.
or 3 NA − NB − W= 0  … (iii)
2 Sol: Apply Newton’s second law in the radial direction
Solving the above three equations, we obtain in each case.
3 3 When the string passes through the ring, the tension
(a) fA = W (b) NA = W (c) NB = W
2 2 in the string is the same in both the parts. Also from
geometry,
P hysi cs | 4.39

BP=3L and AP=5L v2 (10)2


tan=
θ = = 1 ; θ= tan−1 (1)= 45°
4 Rg (10)(10)
T cos=
θ =T mg
5
 3 8 T1
Example 12: A car moves on a horizontal circular road
T + T sin=
θ T 1 + =  T  ... (i)
 5  5 a dv
2 2 of radius R, the speed increases at a rate = a . The
mv mv  … (ii) dt
= =
BP 3L frictional coefficient between the road and the tire is µ .
v2 Now, find the speed at which the car will skid.
Dividing (ii) by (i) =2 mAg sin 60°
3Lg Sol: The net acceleration of the car is the vector sum of
mg 5 centripetal acceleration and tangential acceleration. By
v = 6Lg From (i)
= T = mg . In the second case, Newton’s second law the net force of friction acting on
4 /5 4
the car is equal to mass multiplied by net acceleration.
ABP is an equilateral triangle
Here, at any time t, the speed of the car becomes V,
o o
T1 cos60= mg + T2 cos60 2
 v2 
mg the net acceleration in the plane road is   + a2 .
T1 −=
T2 = 2mg  … (iii) R 
cos60°  

mv 2 9mgL This acceleration is provided by the frictional force. At


T1 sin60° + T2 sin60
= ° = the moment car will slide
r 4L sin60°
2
9mg  v2 
( )
1/ 4
T=
1 + T2 = 3mg  ... (iv) m   + a = µMg ⇒ v= R µ g − a 
2 2 2 2 2
4 sin2 60°
R 
By solving equations (iii) and (iv), we have

5 1
=T1 =
2
mg; T2
2
mg JEE Advanced/Boards
Example 11: A car is moving in a circular horizontal Example 1: In the system of two pulleys connected as
track of radius 10 m with a constant speed of 10 m/s. A shown in the figure, M1 = 4M2 and mass M1 is 20 cm
plumb bob is suspended from roof by a light rigid rod above the ground, whereas mass M2 is lying on the
of length 1 m. What is the angle made by the rod with ground. Find the distance covered by when the system
the track? is released. (g = 10 m / s2 ).
Sol: In the reference frame of the car the bob will
experience a centrifugal force radially outwards. The A
vector sum of the three forces acting on the bob (the
weight, the tension and the centrifugal force) will be T T
equal to zero.
a B
The different forces acting on the bob are shown in T
the Figure. Resolving the force along the length and x₂
perpendicular to the rod, we have
2a
T M₁
20 cm M₂ x₁

2
mv
R Sol: To find the constraint relation between accelerations
 of M1 and M2, measure their distances from fixed pulley
A. Apply Newton’s second law in vertical direction for
each block.
mg M1 = 4M2

mv 2 mv 2 As M1 is heavier, itwill move down with acceleration a


mgcos θ + = sin θ T; mgsin
= θ cos θ and M2 will move upward with acceleration 2a because
R R
4 . 4 0 | Forces and Laws of Motion

the strings around the pulley B will move through half Let a, b and c be the respective accelerations of masses
the distance as compared to that of A. A (8 kg), B (4 kg), and C (16 kg) such that a and b
g are downward and c is upward. Let x1 and x2 be the
∴ M1g − 2T = M1a T − M2g =M2 × 2a or a = .
4 distances of strings from axial line passing through P
Therefore, the time taken for M1 to reach the ground at and Q to the blocks A and B, respectively. Let x3 be the
1 2 1 2 length of the string from the axial line PQ to the center
20 cm distance s =+
ut at =at of the movable pulley. If L is the length of the string,
2 2
then the constraint relation gives
2s 2 × 20 × 4  20 
or
= t = s
=  x1 + x2 + x3 + x3 =L =cons tant
a 10 × 100  100 
dx1 dx2 dx3
16 4 Differentiating + +2 0
=
or= = = 0.45 dt dt dt
100 10
d2 x1 d2 x2 d2 x3
Distance travelled by M2 Differentiating again + +2 0
=
1 1 10 dt2 dt2 dt2
x1 = × (2a) × t2 = × 2 × × (0.4)2 = 0.4m
2 2 4 or a + b −=
2c 0 or a=
+ b 2c.
Velocity of M2 after 0.4 seconds =v=u+2at
As tension T is equal in all the strings as it passes over
10 smooth pulleys, equations for the strings are as follows:
= 0 + 2× × 0.4 ; v = 2ms−1
4
Distance covered by M2 with velocity 2ms-1upwards 8g − T = 8a ... (i)
before coming to rest 2T − 16 g = 16c  ... (ii)
v2 (2)2 4g − T = 4b  ... (iii)
x=
2 = = 0.2 m
2g 2 × 10 a + b = 2c  ... (iv)
From Eqs. (ii) and (iv), we obtain
Distance covered by M2 before coming to rest
=x =x1 + x2 =0.4 + 0.2 =0.6 m 2 T − 16 g =8 × 2c =8a + 8b
By substituting a and b from Eqs (i) and (iii)
Example 2: Masses 4 kg and 8 kg are attached to the
free end of an inextensible light string passing over two 2 T − 16 g = 8g − T + 8g − 2T = 16g − 3T
fixed pulleys as shown in the Figure. The movable pulley 32
carries a mass of 16 kg. Assuming frictionless motion,= 5T 32g = or T g.
5
calculate the acceleration of the three masses.
Now, from Eq (i)
32 32 g
8a = 8g − T = 8g − = ;a =
P Q 5 5 5
From Eq (iii),
a b
x3 T T T 32g −12g
x3 4b = 4g − T = 4g − =
x1 x2 5 5

 g 3g 
 − 
8kg 4kg a+b 5 5  g
c= = ∴ c =−
A c B 2 2 5
16kg ∴ 16 kg and 8 kg go downward and 4 kg go upward.
C

Sol: To find the constraint relation between accelerations Example 3: A block of mass m is pulled up by means
of blocks measure their distances from the fixed pulleys. of a thread up and inclined plane forming an angle α
Apply Newton’s second law in vertical direction for each with the horizontal. The coefficient of friction is equal
block. to µ. Find the angle β which the thread must form with
the inclined plane for the tension of the thread to be
minimum. Also, find the value of minimum tension.
P hysi cs | 4.41

T which is negative.
 ∴ For minimum T,
= β tan−1 µ
The value of Tmin can be found by writing β in terms of
µ.
 1 1
= cos β = ,sin β
2
1+µ 1 + µ2
Sol: Draw the FBD of the block. Apply Newton’s first
law along the perpendicular to the inclined plane and mg ( sin α + µ cos α )
Newton’s second law along the inclined plane for the ∴ Tmin =
cos β + µ sin β
block.
T mg ( sin α + µ cos α ) mg ( sin α + µ cos α )
N = =
T cos  1 µ2 1 + µ2
+
T sin 
 1 + µ2 1 + µ2 1 + µ2

f mg ( sin α + µ cos α )
mg sin  =

 mg cos  1 + µ2
mg

When the body is just about to move up, the force of Example 4: Find the constraint relation in the Figure.
friction f is acting downward. If N is the normal reaction,
the force of friction f is equal to µN. Further, T and mg
can be resolved into rectangular components parallel y₃
and perpendicular to the inclined plane as shown in the
Figure. x₁ y₂
y₁
a₁
∴ T cos
= β mgsin α + µN ....(i)
N+ T=
sin β mgcos α=
or N mgcos α − T sin β m₁

Now, by substituting in Eq.(i), we obtain a₂

T cos
= β mgsin α + µmgcos α − µT sin β m₂

or T cos β + µT=
sin β mg(sin α + µ cos α ) Sol: To find the constraint relation between accelerations
of blocks measure their distances from stationary points.
mg(sin α + µ cos α ) For block m1 measure the distance from fixed pulley on
T=
cos β + µ sin β the wedge. For block m2 measure the distance from the
fixed roof.
For T to be minimum, cos β + µ sin β should be
maximum. Since length of each string is constant
d x1 + (y 2 − y 3 ) + y 2 =
c1  ......(i) ... (i)
∴ (cos β + µ sin β) = 0
dβ y1 − y 2 =
c2 ⇒ 2 y 1 − 2 y 2 =
2c2 ......(ii) ... (ii)
By adding (i) and (ii), we obtain
1+²  (x1 − y 3 + 2 y1 ) = c1 + 2c2

 x1 + 2 y1 = y 3 + c1 + 2c2 = c  .....(iii) ... (iii)


1 (since y 3 = constant )
d2 d2 x1 d2 y1
and (cos β + µ sin β) is negative. Differentiating (iii) w.r.t. t +2 0
=
2
dβ dt2 dt2
sin β −a1 + 2a2 =
0 ⇒ a1 =
2a2  .....(iv) ... (iv)
∴ − sin β + µ cos=
β 0 or =
µ = tan β
cos β
d Example 5: A pendulum is hanging from the ceiling of
Also, ( − sin β + µ cos β) = − cos β + µ sin β
dβ a car having acceleration a0 with respect to the ground.
Find the angle made by the string with the vertical.
4 . 4 2 | Forces and Laws of Motion

Sol: In the reference frame of the car the pendulum A v


bob will experience a pseudo force. For the bob to be in
equilibrium, the vector sum of all the forces acting on it
in the frame of the car should be zero.
B
The situation is shown in the Figure. Suppose that the
mass of the bob is m and string makes an angle θ with
C
the vertical. We shall now proceed based on the car
frame.
 This frame is non-inertial as it has acceleration
Sol: Find the constraint relation between the acceleration
a0 with respect to an inertial frame (the road). Hence, if
of the ring and the block. Measure the distances of ring
we use Newton’s second law we shall have to include a
and the block from the fixed pulley B.
pseudo force.
A A’
Now, consider the bob as the complete system.
P
Then, the forces acting on it are:
(a) T along the string, by the string B
(b) mg downward, by the earth
C
(c) m a0 towards left (pseudo force).
C’
T
(a) Suppose in a small time interval ∆t the ring is
displaced from A to A′ and the block from C to C′. Drop a
a₀ mag perpendicular A′P from A′ to AB. For small displacement
A′B = PB. Since the length of the string is constant,

Mg we have AB + BC = A'B + BC'


(b) or, AP + PB + BC = A'B + BC
(a) or, AP = B'C− BC = CC' (as A'B = PB)
The FBD is shown in the Figure. As the bob is at rest
(remember we are discussing the motion with respect to or, AA'cos θ =CC'
car) the force in (a), (b) and (c) should add to zero. Take AA'cos θ CC'
the X-axis along the forward horizontal direction and or, =
∆t ∆t
the Y-axis along the upward vertical direction.
or, (velocity of the ring) cos θ = (velocity of the block).
The components of the forces along the X-axis give
T sin=
θ − ma0 0 or,
= T sin θ ma0  .....(i) ... (i) (b) If the initial acceleration of the ring is a, that of the
block will be a cos θ . Let T be the tension in the string at
And the components along the Y-axis give
this instant. Consider the block as the system. The forces
T cos
= θ − mg 0 or,= T cos θ mg. ......(ii) acting on the block are
... (ii)
Dividing (i) by (ii) tan θ =a0 / g. (i) Mg downward due to earth, and
Thus, the string makes an angle (ii) T upward due to string.
tan−1 (a0 / g) with the vertical
equation of motion of the block is

Example 6: A smooth ring A of mass m can slide on a Mg-T = Macosθ  ... (ii)
fixed horizontal rod. A string tied to the ring passes over Now, consider the ring as the system. The forces acting
a fixed pulley B and carries a block C of mass M(= 2m) as on the ring are
shown in the Figure. At an instant the string between the
ring and pulley makes an angle θ with the rod. (a) Show (i) Mg downward due to gravity,
that, if the ring slides with a speed v, the block descends (ii) N upward due to the rod,
with speed v cos θ . (b) With what acceleration will the
ring start moving if the system is released from rest with (iii) T along the string due to string.
θ= 30° ? Taking components along the rod, the equation of
motion of the ring is
Tcos θ =ma. 
......(ii) ... (ii)
From (i) and (ii)
P hysi cs | 4.43

ma Motion of: The acceleration is a0, in the horizontal


Mg− = Macos θ
cos θ direction. The forces on m1 are
(a) T by the string (horizontal).
Mgcos θ
or, a= (b) m1 g by the earth (vertically downward) and
m + Mcos2 θ
(c) N by the table (vertically upward).
2
Putting θ= 30° , M = 2 m and g = 9.8 m / s ; In the horizontal direction, the equation is
2
therefore, A = 6.78 m / s T = m1a 0  ... (ii)
Motion of: Acceleration is a0 − a in the downward
Example 7: Three blocks of A direction. The forces on m2 are
masses m1, m2 and m3 are m₁
connected as shown in the (a) m2 g downward by the earth and
Figure. All the surfaces are B
(b) T′ = T/2 upward by the string.
frictionless and the string
and the pulleys are light. m₂ Thus m2g − T 2= m2 (a0 − a)  .....(iii) ... (iii)
Motion of m3 : Acceleration is ( a0 + a) in the downward
m₃
Find the acceleration of m1.
Sol: Draw the FBD of all the blocks and the pulley B. direction. The forces on are
The acceleration of pulley B is same in magnitude as the (a) m3 g downward by the earth and
acceleration of m1. In the frame of pulley B blocks m2
and m3 will experience pseudo forces. (b) T′= T/2 upward by the string.

Suppose the acceleration of m1 is a0 toward the right. Thus m3g − T 2= m3 (a0 + a)  .....(iv) ... (iv)
That will also be the downward acceleration of the We want to calculate a0 , so we shall eliminate T and a
pulley B because the string connecting m1 and B is from (ii), (iii), and (iv).
constant in length. This implies that the decrease in
the separation between m2 and B equals the increase Putting T from (ii) in (iii) and (iv),
in the separation between m3 and B. Therefore, the m2g − m1 a0 2 m1a0
upward acceleration of m2 with respect to B equals the a0 − a = = g−
m2 2m2
downward acceleration of m3 with respect to B. Let this
acceleration be a. m3g − m1 a0 2 m1a0
and a0 + a = = g−
The acceleration of with respect to the ground = a0 − a m3 2m3
(downward) and the acceleration of with respect to the
ground = a0 + a (downward). ma  1 1 
2g − 1 0
Adding, 2a0 =  + 
These accelerations will be used in Newton’s laws. Let 2  m2 m3 
the tension be T in the upper string and T′ in the lower
ma  1 1 
string. Consider the motion of the pulley B. or, g− 1 0 
a0 = + 
4  m2 m3 
The forces on this light pulley are
(a) T upward by the upper string and  m  1 1 
or, a0 1 + 1  +  = g
(b) 2T′ downward by the lower string.  4  m2 m3  

As the mass of the pulley is negligible, g


or, a0 =
2T′ – T = 0 Giving T′= T/2.  ... (i)  m1  1 1 
1 +  + 
a₀  4  m2 m3  
m₁ A

T
Example 8: All the surfaces shown in the figure. are
assumed to be frictionless. The block of mass m slides
B
on the prism which in turn slides backward on the
T’ horizontal surface. Find the acceleration of the smaller
m₂ block with respect to the prism.
a ₀-a m₃
a ₀+a
4 . 4 4 | Forces and Laws of Motion

m Example 9: A block of mass 0.4 kg is attached to a


vertical rotating spindle of length 1.6 m by two springs
M each of length 1 m of equal lengths as shown in the
Figure. The period of rotation is 1.2 seconds. Find the
tension in the springs.
Sol: Draw the FBD of both the blocks. In the reference R
frame of block M, the block m will experience pseudo C T2sin 
force. Apply Newton’s second law on the block m along T₂
the inclined plane and Newton’s first law along the 0.8m 1.0m
perpendicular to the inclined plane. For block M apply
0.4 kg
Newton’s second law along the horizontal. B
A
Let the acceleration of the prism be in the backward
T₁
direction. Consider the motion of the smaller block from
the frame of the prism. mg +T1sin 
T1cos +T2cos 
The forces on the block are
Sol: The sum of the horizontal components of tensions
(i) N normal force, in the two springs will provide the necessary centripetal
(ii) mg downward (gravity), and acceleration to the block. The vector sum of the vertical
components of tensions in the two springs will balance
(iii) m a0 forward(pseudo). the weight of the block.
N
N’ Let T1 and T2 be the tension in the springs when these
ma₀ springs subtend an angle θ each with the horizontal
N
direction. Let AB = R be the radius of circular path
mg
traversed by mass B in the horizontal plane.
mg
(a) (b) R =AB = 1 − (0.8)2 = 1 − 0.64 = 0.36 =0.6
The block slides down the plane. Components of the AC 0.8
forces parallel to the incline give sin=
θ = = 0.8 m
BC 1
ma0 cos θ + mgsin θ = ma
AB 0.6
or
= a a0 cos θ + gsin θ  ......(i) ... (i) cos=
θ = = 0.6 m
BC 1
Components of the force perpendicular to the incline 2π 2π π
Angular velocity = ω = = =
give N+ ma0=
sin θ mgcos θ.  ...(ii) ... (ii) T 1.2 0.6
Now, consider the motion of the prism from the lab Resolving T1 and T2 into rectangular component
frame. No pseudo force is needed as the frame used is
T2 sin
= θ T1 sin θ + mg
inertial. The forces acting now are
(i) Mg downward, (T1 + T2 )cos θ= mω2R .....(i)

(ii) N normal to the incline (by the block), and T2cos θ= mω2R − T1cos θ .....(ii)
(iii) N′ upward (by the horizontal surface). Multiply (i) by cos θ ,
Horizontal components give, T2 sin θ cos θ= T1 sin θ cos θ + mgcos θ
Nsin θ Ma0=
= or, N Ma0 sin θ.  .....(iii) ... (iii) Multiply (ii) by sin θ ,
Ma0 T2 sin θ cos θ =mω2 R sin θ − T1 sin θ cos θ
Replacing in (ii) + Ma0=
sin θ mgcos θ
sin θ Adding, 2T2 sin θ cos θ= mω2 R sin θ + mgcos θ
mgsin θ cos θ
or, a0 =
M + msin2 θ mω2 R sin θ mgcos θ
=2T2 +
sin θ cos θ sin θ cos θ
mgsin θ cos2 θ (M+ m)gsin θ
From (i),=a0 + gsin θ = 0.4 × π2 × 0.6 0.4 × 10 0.4 × π2
M + msin θ2 ω2 R mg
M + msin2 θ = m= + = +5
+ 0.8
cos θ sin θ (0.6)2 × 0.6 (0.6)2
P hysi cs | 4.45

2T
=2 10.97 +=
5 15.97 ∴=
T2 7.99 N ≈ 8 N = cos α + µ2 cos=
α cos α(1 + µ2 )
Subtracting the above mentioned terms, 1 + µ2 1 + µ2 1 + µ2
= = = = 1 + µ2
2
mω R mg sec α 1 + tan2 α 1+µ 2
2T1
= − = 10.97 −=
5 5.97
cos θ sin θ
mgsin θ + µ mgcos θ
T1 2.99 ≈ 3 N
= ∴ Tmin =
1 + µ2
Example 10: A block of mass m is pulled by means of
T
a thread up an inclined plane forming an angle θ with Example 11: A metal ring of
the horizontal. The coefficient of friction is µ . Find the mass m and radius R is placed
inclination of the thread with the horizontal so that on a smooth horizontal table A
tension in the thread is minimum. What is the value of and is set rotating about its own
/2
the minimum tension? axis in such a way that each part O
/2 C
of the ring moves with velocity
Sol: Draw the FBD of the block. Apply Newton’s second
v. Based on the above facts, find
law along the direction of the incline and Newton’s first B
the tension in the ring.
law along the direction perpendicular to the incline.
Let the mass moves up the plane with acceleration a. Sol: Each small part of the ring T
will experience a centrifugal
Writing the equation of motion, we obtain
force radially outwards. So the ring will tend to expand,
R + T=
sin α mgcos θ i.e. the radius and circumference will tend to increase. By
=R mgcos θ − T sin α virtue of its elasticity the ring will oppose its expansion.
 … (i)
So each part of the ring will experience a force of pull or
T cos α − mgsin θ − f = ma  ... (ii) tension from the other part.
where t is the force of friction Consider a small part ACB of the ring that subtends an
f = µ(mgcos θ − Tsin α )  … (iii) angle ∆θ at the center as shown in the Figure. Let the
tension in the ring be T.
Substituting the value of f from Eq (iii)
The forces on this elementary portion ACB are:
in Eq (ii) T cos α − mgsin θ − µ mgcos θ + µ Tsin α = ma
(i) Tension T by the part of the ring left to A
T(cos α + msin α=
) ma + mgsin θ + µ mgcos θ
(ii) Tension T by the part of the ring right to B
ma + mgsin θ + µ mgcos θ  … (iv)
T= (iii) Weight ( ∆m )g
cos α + µ sin α
(iv) Normal force N by the table
For T to be minimum (cos α + µ sin α ) should be
d As the elementary portion ACB moves in a circle of
maximum (cos α + µ sin α ) = 0 radius R at constant speed v, its acceleration toward the

( ∆m) v 2
d2 centre is Resolving the forces along the radius
(cos α + µ sin α ) = − ve R
dα 2 CO
2
d  ∆θ   ∆θ  v  … (i)
(cos α + µ sin α ) = − sin α + mcos α = 0 T cos  90° −  + T cos  90° −  = ∆m R
dα  2   2 
= tan−1 (µ )
µ tan α α
=
∆θ v2
2
d 2T sin = ∆m  ... (ii)
It can be shown that is negative. 2 R
dα2 Thus the length of the part ACB = R ∆θ . The mass
T will have minimum value when a = 0 and m
per unit length of the ring is
=α tan−1 (µ ) . From Eq. (iv) 2πR
mgsin θ + µ mgcos θ ∴ Mass of this portion ACB, ∆m =R∆θm m∆θ
Tmin = =
cos α + µ sin α 2πR 2π
cos α + µ sin=
α cos α + µ(µ cos α )
4 . 4 6 | Forces and Laws of Motion

Putting the value of ∆m in (ii)  


 ∆θ 
∆θ m∆θ v 2  2  mv 2
2T sin Since  is equal to 1; T =
=   2πR
2 2π R   sin  ∆θ   
 2 
   
 
 ∆θ 
2
mv  2 
∴T = 
2πR   ∆θ   
  sin    

   2  

JEE Main/Boards

Exercise 1 Q.12 State and explain Newton’s third law of motion.


Give at least two Illustrations.
Forces and Laws of Motion
Q.13 Discuss the apparent weight of a man in a lift/
Q.1 What is meant by law of inertia? elevator.

Q.2 State the laws of motion. Q.14 Two bodies of masses 11 kg and 11.5 kg are
connected by a long light string passing over a
Q.3 A cricket player lowers his hands while catching a smooth pulley. Calculate velocity and height ascended/
ball. Why? descended by each body at the end of 4s.
Y
Q.4 An impulsive force of 100N acts on a body for 1 s. Q
What is the change in its linear momentum?
X
O P
Q.5 A force of 5N changes the velocity of a body from
10 ms-1 to 20 ms-1 in 5 sec. How much force is required
to bring about the same change in 2 sec? Q.15 A rope of mass 0.5 kg is pulling a block of mass
10 kg under the action of force of 31.5 N. If the block
Q.6 State and explain Newton’s first law of motion. is resting on a smooth horizontal surface, calculate the
force of reaction exerted by the block on the rope.
Q.7 What are the three types of inertia? Give at least
two examples of each type. Q.16 Two bodies of masses 4 kg and 3 kg respectively
are connected by a light string passing over a smooth
Q.8 State and explain Newton’s first law of motion. frictionless pulley. Calculate the acceleration of the
Hence deduce the relation F = ma, where the symbols masses and tension in the string.
have their usual meaning.
Q.17 Two bodies whose masses are m1=50 kg and
Q.9 Define absolute and gravitational units of force. m2=50 kg are tied by a light string and are placed on
What are the dimensions of force? a frictionless horizontal surface. When m1 is pulled by
a force F, an acceleration of 5 ms-2 is produced in both
Q.10 Mention some of the consequences of the the bodies. Calculate the value of F. What is the tension
Newton’s second law of motion. in the string 1?

Q.11 Explain the term ‘impulse’. Discuss some of the


applications of this concept.
P hysi cs | 4.47

Q.18 See Figure where in Q.26 A cyclist is riding with a speed of 27 kmh-1. As he
1m
a mass of 6 kg is approaches a circular turn on the road of radius 80.0 m,
T₁
suspended by a rope 50N
he applies brakes and reduces his speed at a constant
P
of length 2 m from the rate of 0.5 ms-1 per second. Find the magnitude of the
ceiling. A force of 50N 1m net acceleration of the cyclist.
T₂
in the horizontal
direction is applied at W Q.27 A particle moves in a circle of radius 4.0 cm
 
midpoint P of the rope, clockwise at constant seed of 2 cms-1. If x and y
60N
as shown. What is the are unit acceleration vectors along X-axis and Y-axis,
(a)
angle the rope makes respectively, find the acceleration of the particle at the
with the vertical in instant half-way between P and Q in the Figure.
T₂
equilibrium?
T₁
(take g = 10 ms–2) W Q.28 A cyclist is riding with a speed of 36 kmh-1. As he
P 50N
Neglect mass of the approaches a circular turn on the road of radius 140 m,
rope. T₂ he applies brakes and reduces his speed at the constant
60N rate of 1 ms−2 . What is the magnitude and direction of
(b) (c) the net acceleration of the cyclist on the circular turn?

Q.19 A body builder exerts a force of 150N against a


bull worker and compresses it by 20 cm. Calculate the Exercise 2
spring constant of the spring in the bull worker.
Forces and Laws of Motion
Q.20 A lift of mass 2000 kg is supported by thick steel Single Correct Question
ropes. If maximum upward acceleration of the lift be
1.2 m / s2 , and the breaking stress for the ropes be Q.1 A block of mass 10kg is suspended through two
2.8 × 108 Nm−2 , what would be the minimum diameter light spring balances as shown in given Figure.
of the rope?

Q.21 A car of mass one metric ton travelling at 32 m/s


dashes into rear of a truck of mass 8000 kg moving in
the same direction with the velocity of 4 m/s. After the
collision, the car bounces backward with the velocity 8
m/s. What is the velocity of the truck after the impact?

Q.22 The force on a particle of mass 10 g is (10i+5j)N. 10kg

If it starts from rest, what would be its position at time (A) Both the scales will read 10 kg
t = 5s?
(B) Both the scales will read 5 kg.
Q.23 A projectile is fired vertically from the earth’s surface (C) The upper scales will read 10 kg and the lower zero.
with an initial velocity of 10 km/s. Neglecting atmospheric
retardation, how far above the surface of the earth would (D) The readings may be anything but their sum will be
it go? Take the earth’s radius as 6400 km. 10 kg

Q.24 Two balls of mass m each are hung side by side Q.2 A block is kept on the floor of an elevator at rest.
two long threads, each of length l. If the distance The elevator starts descending with an acceleration of
between the upper end is r then find the distance r′ 12 m / s2 . Find the displacement of the block during the
2
between the centres of the ball in terms of g, r, l and m. first 0.2 s after the start. Take g=10 m / s .
(A) 10 cm (B) 20 cm (C) 30 cm (D) 40 cm
Circular Motion
Q.3 A body of mass m is kept on a rough horizontal
Q.25 Calculate the centripetal acceleration of a point on surface (friction coefficient=µ). A person is trying to pull
the equator of earth due to the rotation of earth about the body by applying a horizontal force but the body is
its own axis. not moving. The force by the surface on the body is F
Radius of earth = 6400 km. where
4 . 4 8 | Forces and Laws of Motion

(A) F=mg (B) F=µmg Q.9 Two masses m and m’ are tied with a thread passing
over a pulley. M’ is on a frictionless horizontal surface
(C) mg ≤ F ≤ mg 1 + µ2 (D) mg ≥ F ≥ mg 1 − µ2 and m is hanging freely. If acceleration due to gravity
is g, the acceleration of m’ in this arrangement will be
Q.4 Which of the following case correctly represents the (A) g (B) mg/(m + m’)
applied force on a string under tension. End of string is
represented with dot. (C) mg/m’ (D) mg/(m-m’)
F FF F
Q.10 A body of mass 60 kg is dragged with just enough
(A) (A) (B) (B) force to start moving on a rough surface with coefficients
F F of static and kinetic frictions 0.5 and 0.4 respectively.
F F
On continuing (g=9.8 m/s2) the same force what is the
acceleration:
(C) (C) (D) (D)
(A) 0.98 m/s2 (B) 9.8 m/s2
(C) 0.54 m/s2 (D) 5.292 m/s2

Q.5 A balloon is descending at a constant acceleration


Q.11 Which of the following represents 2nd law of
a. The mass of the balloon is M.
motion most correctly.
When a mass m is released from the balloon it starts   
 dv
rising with acceleration a. Assuming that volume does (A) F = ma (B) F = m
dt
not change when the mass is released, what is the value  dp  
of m? [Assume same upward buoyant force] (C) F = (D) F = mv
dt
2a a+ g
(A) M (B)  M
(a+ g)  2a  Q.12 Two objects A and B are thrown upward
2a Ma simultaneously with the same speed. The mass of A
(C) (D) is greater than the mass of B. Suppose the air exerts
(a+ g)M a+g
a constant and equal force of resistance on the two
bodies.
Q.6 A small cart with a sphere suspended from ceiling
by a string is moving up an inclined plane at a speed V. (A) The two bodies will reach the same height.
The direction of string supporting the sphere is (B) A will go higher than B
(A) Vertical (C) B will go higher than A
(B) Horizontal
(D) Any of the above three may happen depending on
(C) Perpendicular to the inclined plane the speed with which the objects are thrown.
(D) None of these
Q.13 A heavy uniform chain party lies on a horizontal
Q.7 The pulleys and strings shown in the figure are table. If the coefficient of friction between the chain and
smooth and of negligible mass. For the system of remain the table surface is 0.25, then the maximum fraction of
in equilibrium, the angle θ should be the length of the chain that can hang over edge of the
table is
(A) 20% (B) 25% (C) 33% (D) 15%.

Q.14 An insect crawls up hemispherical surface very


2m
m m slowly as shown in Figure. The coefficient of friction
between insect and surface is 1/3. If
(A) 0o
(B) 30o (C) 45o (D) 60o the line joining the centre of the

hemispherical surface to the insect
Q.8 While walking on ice, one should take small steps makes an angle α with the vertical,
to avoid slipping. This is because smaller steps ensure the max. Possible value of α is given
by
(A) Larger friction (B) Smaller friction
(A) cot α=3 (B) sec α=3 (C) cosec α=3 (D) None
(C) Larger normal force (D) Smaller normal force
P hysi cs | 4.49

Q.15 When a bird of weight W alights on a stretched Q.21 A uniform chain of length  is placed on a rough
wire, the tension T in the wire may be: table with length n  hanging over the edge (n<  ). If
(A)>W/2 (B)=W (C)<W (D) None of these. the chain just begins to slide off the table by itself from
this position, the coefficient of friction between chain
and table is
Q.16 A block of mass 3kg is at rest 3kg 1 n 1 1−n
on a rough inclined plane as shown (A) (B) (C) (D)
n 1−n n+1 1+n
in the Figure. The magnitude of net
force exerted by the surface on the 30°
block will be Circular Dynamics
(A) 26N (B) 19.5N (C) 10N (D) 30N Single Correct Question

Q.17 With what minimum acceleration can a fireman Q.22 A particle moves in a circle of radius R with a
slides down a rope whose breaking strength is two third constant speed under a centripetal force F. The work
of his weight? done F in completing a full circle is:

(A) g/2 (B) 2g/3 (C) g/3 (D) 3g/4 ( )


(A) Mv 2 / R 2πR (B) πR 2F
(C) 2πRF (D) zero
Q.18 Forces of 30N, 40N and 50N act along the sides
  
AB ,BC and CA of an equilateral triangle ABC. The triangle Q.23 When a particle is rotated in a vertical plane with
is of mass 0.5 kg and kept in a constant angular velocity magnitude of centripetal
A
vertical plane as shown in the 50N force is:
Figure. With the side AB vertical. 30N (A) Maximum at highest point
The net vertical force acting on C
W
the triangle will be (g=10 m/s2) (B) Maximum at lowest point
40N
B
(C) Same at all points
(A) 125 N upwards (B) 5 N downwards (D) Zero
(C) 10 N upwards (D) 25 N downwards
Q.24 In uniform circular motion, the quantity that
Q.19 A trolley is being pulled up an incline plane by a remains constant is:
man sitting on it (as shown in Figure). He applies a force (A) Linear velocity (B) Centripetal force
of 250N. If the combined mass of the man and trolley is
100 kg, the acceleration of trolley will be [ sin 15° =0.26 ] (C) Acceleration (D) Speed

Q.25 Two particles of equal masses are revolving in


250N
circular paths of radii r1 and r2 respectively with the
same speed. The ratio of their centripetal forces is:
5° 2 2
r2 r2 r  r 
(A) (B) (C)  1  (D)  2 
(A) 2.4 m/s2 (B) 9.4 m/s2 r1 r1 r  r 
 2  1
(C) 6.9 m/s2 (D) 4.9 m/s2
Q.26 A 500kg car takes a round turn of radius 50m with
Q.20 A body is placed on a rough inclined plane of a velocity of 36km/hr. The centripetal force is:
inclination. As the angle θ is increased from 0° to 90° , (A) 250 N (B) 750 N (C) 1000 N (D) 1200 N
the contact force between the block and plane
(A) Remains constant Q.27 A body of mass 5kg is moving in a circle of
(B) First remains constant then decreases radius 1m with an angular velocity of 2 radian/sec. The
(C) First decreases then increases centripetal force is:
(D) First increases then decreases (A) 10 N (B) 20 N (C) 30 N (D) 40 N
4 . 5 0 | Forces and Laws of Motion

Q.28 A motorcycle is going on an overbridge of radius Previous Years’ Questions


R. The driver maintains a constant speed. As the
motorcycles is ascending on the overbridge, the normal Forces and Laws of Motion
force on it:
(A) Increase Q.1 A ship of mass 3 × 107 kg initially at rest, is pulled by
a force of 5 × 10 4 N through a distance of 3 m. Assuming
(B) Decreases
that the resistance due to water is negligible, the speed
(C) Remains constant of the ship is (1980)
(D) First increases then decreases. (A) 1.5 m/s (B) 60 m/s
(C) 0.1 m/s (D) 5 m/s
Q.29 If a particle of mass m is moving in a horizontal
(
circle of radius r with a centripetal force −k / r 2 , the ) Q.2 A block of mass 2 kg rests on a rough inclined plane
total energy of the particle is:
k k 2k 4k making an angle of with the horizontal. The coefficient
(A) − (B) − (C) − (D) − of static friction between the block and the plane is 0.7.
2r r r r
The frictional force on the block is (1980)
Q.30 A person with his hands in his pocket is skating on (A)9.8N (B)0.7 × 9.8 × 3N
ice at the rate of 10m/s and describes a circle of radius (C)9.8 × 3N (D)0.7 × 9.8N
50m. What is his inclination to the vertical:
(A) tan−1 (1 / 2 ) (B) tan−1 (1 / 5 ) Q.3 A car is moving in a circular horizontal track of
(C) tan−1
(3 / 5) (D) tan (1 / 10 ) −1 radius 10 m with a constant speed of 10 m/s. A plumb
bob is suspended from the roof of the car by a light
Q.31 A ball tied to a string (in vertical plane) is swinging rigid rod. The angle made by the rod with vertical is
in a vertical circle. Which of the following remains (Take g = 10m / s2 )  (1992)
constant during the motion? (A) Zero (B)30o (C)45o (D)60o
(A) Tension in the string
Q.4 A block of mass 0.1 kg is held against a wall applying
(B) Speed of the ball
a horizontal force of 5N on the block. If the coefficient
(C) Centripetal force of friction between the block and wall is 0.5, the
(D) Gravitational force on the ball magnitude of the frictional force acting on the block is
(1994)
Q.32 A heavy particle hanging vertically from a point by (A) 2.5 N (B) 0.98 N (C) 4.9 N (D) 0.49 N
a light inextensible string of length l is started so as to
make a complete revolution in a vertical plane. The sum Q.5 A long horizontal rod has a bead which can slide
of the tension at the ends of any diameter: along its length and initially placed at a distance L from
(A) First increase then decreases one end A of the rod. The rod is set in angular motion
about A with a constant angular acceleration α. If the
(B) Is constant coefficient of friction between the rod and bead is µ,
(C) First decrease then increases and gravity is neglected, then the time after which the
bead starts slipping is (2000)
(D) Decreases continuously
µ µ 1
Q.33 In a circus, stuntman rides a motorbike in a circular (A) (B) (C) (D) infinitesimal
a a µa
track of radius R in the vertical plane. The minimum
speed at highest point of track will be: Q.6 A small block is shot into each of the four tracks as
shown below. Each of the tracks rises to the same height.
(A) 2gR (B) 2gR (C) 3gR (D) gR
The speed with which the block enters the track is the
same in all the cases. At the highest point of track, the
normal reaction is maximum in (2001)
P hysi cs | 4.51

Q.10 What is the maximum value of force F such that


the block shown in the arrangement. does not move?
(2003)
F m= 3kg
v v 60° = 1
2 3
(A) (B)

(A) 20 N (B) 10 N (C) 12 N (D) 15 N

Q.11 A block of mass m is at rest under the action of


v v
force F against a wall as shown in Figure. Which of the
(C) (D) following statement is incorrect?  (2005)

Q.7 An insect crawls up a hemispherical surface very F


slowly (see the Figure). The coefficient of friction between
the surface and insect is 1/3. If the line joining the
center of the hemispherical surface to the insect makes (A) f=mg (where f is the frictional force)
an angle α with the vertical, the maximum possible (B) F=N (where N is the normal force)
value of α is given (2001)
(C) F will not produce torque

(D) N will not produce torque

Q.12 System shown in Figure is in equilibrium


(A) cot α = 3 (B) tan α = 3
and at rest. The spring and string are massless.
(C) sec α = 3 (D) cosec α = 3 Now the string is cut. The acceleration of
mass 2m and m just after the string is cut will
be (2006)
Q.8 A string of negligible mass going 2m
over a clamped pulley of mass m (A) g/2 upwards, g downwards
m
supports a block of mass M as shown in (B) g upwards, g/2 downwards
the Figure. The force on the pulley by m
the clamp is given by (2001) (C) g upwards, 2g downwards
M (D) 2g upwards, g downwards

Q.13 A piece of wire is bent in the shape of a parabola


(A) 2Mg (B) 2mg
(y-axis vertical) with a bead of mass m on it. The bead can
slide on the wire without friction. It stays at the lowest
(C) (M+ m)2 + m2g (D)  (M+ m)2 + M2  g
  point of the parabola when the wire is at rest. The wire
is now accelerated parallel to the x-axis with a constant
acceleration a. The distance of the new equilibrium
Q.9 The pulleys and strings position of the bead, where the bead can stay at rest
shown in the Figure. are with respect to the wire, from the y-axis is (2009)
smooth and of negligible mass. a a 2a a
(A) (B) 2gk (C) (D)
For the system to remain in gk gk 4gk
equilibrium, the angle θ should
be (2001) 2m
m m
Q.14 A block of mass m is on an inclined plane of angle
θ . The coefficient of friction between the block and
plane is µ and tan θ > µ. The block is held stationary by
(A) Zero (B)30o (C)45o (D)60o applying a force P parallel to the plane. The direction of
force pointing up the plane is taken to the positive. As
P is varied from
4 . 5 2 | Forces and Laws of Motion

Fo Fo
m
P mb

v(t) v(t)
(A) (B)
=P1 mg ( sin θ − µ cos θ ) =
to P2 mg ( sin θ + µ cos θ ) the t t

frictional force f versus P graph look like (2010) Fo Fo


mb mb
f f
P₂ v(t) v(t)
(A) (B)
P₁ P P₁ P₂ P (C) (D)
t t

f f

P₁ P1 P2 Q.19 A block of mass m is placed on a surface with a


(C)
P₂ P
(D) vertical cross section given by y = x3/6. If the coefficient
P
of friction is 0.5, the maximum height above the
ground at which the block can be placed without
Q.15 A reference frame attached to the earth (1986) slipping is: (2014)

(A) Is an inertial frame by definition. 1 1 1 2


(A) m (B) m (C) m (D) m
3 2 6 3
(B) Cannot be an inertial frame because the earth is
revolving round the sun.
Q.20 Given in the figure are two blocks A and B of weight
(C) Is an inertial frame because Newton’s law are 20 N and 100 N respectively. These are being pressed
applicable in this frame. against a wall by a force F as shown. If the coefficient of
(D) Cannot be an inertial frame because the earth is friction between the blocks is 0.1 and between block B
rotating about its own axis and the wall is 0.15, the frictional force applied by the
wall on block B is (2015)
Circular Dynamics (A) 80 N (B) 120 N (C) 150 N (D) 100 N

Q.16 A car is moving in a circular horizontal track of Q.21 A point particle of mass m, moves along the
radius 10m with a constant speed of 10m/s. A plumb uniformly rough track PQR as shown in the figure. The
bob is suspended from the roof of the car by a light coefficient of friction, between the particle and the
rigid rod. The angle made by the rod with the vertical is rough track equals µ. The particle is released, from rest,
(Take g=10/s2) (1992) from the point P and it comes to rest at a point R. The
energies, lost by the ball, over the parts, PQ and QR, of
(A) Zero (B) 30
o
(C) 45o
(D) 60o
the track, are equal to each other, and no energy is lost
when particle changes direction from PQ to QR. (2016)
Q.17 A ball of mass (m) 0.5 kg is attached
to the end of a string having length (L) The values of the coefficient of friction µ and the distance
0.5m. The ball is rotated on a horizontal L x(=QR), are, respectively close to :
circular path about vertical axis. The (A) 0.2 and 3.5 m (B) 0.29 and 3.5 m
maximum tension that the string can
bear is 324 N. The maximum possible (C) 0.29 and 6.5 m (D) 0
m
value of angular velocity of ball (in rad/s)
3kg − ms−1 (2011)
(A) 9 (B) 18 (C) 27 (D) 36

Q.18 A particle of mass m is at rest at the origin at time t


= 0. It is subjected to a force F (t) = F0e–bt in the x direction.
Its speed v(t) is depicted by which of the following
curves? (2012)
P hysi cs | 4.53

JEE Advanced/Boards

Exercise 1 Q.4 The rear side of a truck is open and a box of 40 kg


mass is placed 5 m away from the open end as shown in
Forces and Laws of Motion Figure. The co-efficient of friction between the box and
the surface below it is 0.15. On a straight road, the truck
Q.1 A man of mass 70 kg stands on weighting scale in a starts from rest and accelerates with 2 ms−2 At what
lift which is moving distance from the starting point does the box fall of the
truck? (Ignore the size of the box).
(a) Upwards with a uniform speed of 10 ms−1 .
(b) Downwards with a uniform acceleration of 5 ms−2 .
(c) Upwards with a uniform acceleration of 5 ms−2 .
(d) What would be the reading if the lift mechanism
failed and it hurtled down freely under gravity?
What would be the readings on the scale in each case? Q.5 A helicopter of mass 1000 kg rises with a vertical
acceleration of 15 ms−2 . The crew and the passenger
x(m) weigh 300 kg. Give the magnitude and direction of the
(a) Force on the floor by the crew and passengers.
3 A (b) Action of the rotor of the helicopter on the
surrounding air.
0 4 t(s) (c) Force on the helicopter due to the surrounding air.
Q.2 A block of mass 25 kg is raised by a 50 kg man in two
different ways as shown in Figure. What is the action on Q.6 A block of mass 15 kg is placed on a long trolley.
the floor by the man in the two cases? If the floor yields The co-efficient of static friction between the block and
to a normal force of 700 N, which mode should man trolley is 0.18. The trolley accelerates from rest with
adopt to lift the block without the floor yielding? 0.5 ms-2 for 20 s and then moves with uniform velocity.
Discuss the motion of the block viewed by (a) a stationary
observer on the ground. (b) an observer moving with
the trolley.

Q.7 Both the springs shown in the Figure are un-


stretched. If the block is displaced by a distance x and
released, what will be the initial acceleration?
k₁ m k₂
25kg 25kg
(a) (b)
Q.8 Three equal weights of 2 kg each are hanging
Q.3 A monkey of mass 40 kg climbs on a over the frictionless pulley. Find the acceleration of the
rope which can stand a maximum system and tension of the string connecting weights A
tension of 600 N. In which of the and B. (g=10 m/s2)
following cases will the rope break: the
monkey Q.9 Find the tension in OB and AB in the given Figure.
(a) Climbs up with an acceleration of Also, calculate the tension in OB when just after the
6ms-2 string AB is burnt. O
(b) Climbs down with an acceleration of
4ms-2
(c) Climbs up with a uniform speed of 5ms-2 m
A
B
(d) Falls down the rope nearly freely under gravity.
4 . 5 4 | Forces and Laws of Motion

Q.10 A man of mass m has fallen into a ditch of width Q.15 At the moment t=0 the force F=at is applied to a
d and two of his friends are slowly pulling him out small body of mass m resting on a smooth horizontal
using a light rope and two fixed pulleys as shown in plane (a) is constant). The permanent direction of this
Figure. Show that the force (assumed equal for both the force forms as angle α with the horizontal. Find (a) The
friends) exerted by each friend on the rope increases as velocity of the body at the moment of its breaking off
the man move up. Find the force when the man is at a the plane; (b) The distance traversed by the body up to
depth h. this moment.
F

h
m

Q.16 Two identical block A and B each of mass M are


d connected through a light inextensible string. Coefficient
of friction between blocks and surfaces are µ as shown.
Q.11 The elevator shown in the
2m/s² Initially string is relaxed in its normal length. Force F is
Figure is descending with an
applied on block A as shown. Find the force of friction
acceleration of 2m/s2. The mass of A on blocks and tension in the string.
block A is 0.5 kg. What force es B
exerted by the block A on the block
Q.17 In the Figure block A is one fourth the length of
B?
the block B and there is no friction between block B and
surface on which it is placed. The coefficient of sliding
Q.12 The force of buoyancy exerted by the atmosphere
friction between A and B. Block C and block A have the
on a balloon is B in the upward direction and remains
same mass and mass of B is four times mass of A. when
constant. The force of air resistance on the balloon acts
the system is released, calculate the distance the block
opposite to the direction of velocity and is proportional
B moves when only three fourth of block A is still on the
to it. The balloon carries a mass M and is found to fall
block B.
down near the earth’s surface with a constant velocity v.
How much mass should be removed from the balloon A
so that it may rise with a constant velocity v? B

l C
Q.13 Two touching bars 1 and 2 are placed on an
inclined plane forming an angle α with the horizontal
shown in Figure. The masses of the bars are equal to Q.18 The inclined plane of forms an angle α=30o with
m1 and m2 and the coefficients of friction between the horizontal. The mass ratio. The coefficient of friction
the inclined plane and the bars are equal to k1 and k 2 between the body and inclined plane is equal to k-0.10.
respectively, with k1 > k 2 . The masses of the pulley and the threads are negligible.
Find the magnitude and the direction of acceleration of
Find (a) The force of interaction 
the body m2 when the system of masses starts moving.
of the bars in the process of 2
motion; (b) The minimum value 1 m1
of the angle α at which the bars
 m2
start sliding down.


Q.14 A small body A starts sliding down from the top Q.19 As shown in the Figure blocks of masses M/2, M
of fixed wedge (as shown in the Figure) whose base is and M/2 are connected through a light string as shown,
equal to l=2.10 m. The coefficient of friction between pulleys are light and smooth. Friction is only between
the body and the wedge surface is k=0.140. At what block C and floor. System is released from rest. Find the
value of the angle α will the time of sliding be the least? acceleration of blocks A, B and C and tension in the
string. C
A 2
B M/
 = tan
M 2
 A
l 2
M/

P hysi cs | 4.55

Q.20 On a smooth inclined plane of angle α is placed on between the bar and electric motor is equal to l. When
in such a way that the upper wedge face is horizontal. the motor is switched on, the bar, whose mass is twice
On this horizontal face is placed a block of mass m. Find as great as that of the other body, starts moving with
the resultant acceleration of the block in subsequent a constant acceleration w. How soon will the bodies
motion. collide?
m
Wedge  Q.26 Two particle of equal masses m and m are
M
connected up a light string of length 2l. A constant force
 F is applied continuously at the middle of the string,
always along the perpendicular bisector of the line
joining the two particles. Show that when the distance
Q.21 In the system shown in Figure. = mA 4m,m= 3m
B between the particles is 2x, the acceleration of approach
and mC = 8m . Friction is absent everywhere. String is
fx
light and inextensible. If the system is released from rest of particles is .
1
find the acceleration of each block.
m(2 – x2 ) 2
B
Q.27 Determine the acceleration of bodies A and B and
A
the tension in the cable due to application of the 300 N
C
force. Neglect all friction and the masses of pulleys.

Q.28 Two blocks A and B having masses m1 = 1 kg and


Q.22 Find the accelerations of m2 = 4 kg are arranged as shown in Figure. The pulleys
rod A and wedge B in the P and Q are light and frictionless. All blocks are resting
arrangement shown in the on the horizontal floor and pulleys are held such that
Figure. If the ratio of the mass strings remain just taut. At moment t=0, a force F=30N
A
of the wedge of that of the starts acting on the pulley P along vertically upward
rod equals, and the friction B direction as shown in the Figure: Determine
between all contact surfaces 
(a) The time when blocks A and B lose contact with
is negligible.
ground,
Q.23 A particle of mass m is pulled by means of a (b) The velocity of A when B loses contact with ground,
thread up an inclined plane forming an angle α with (c) The height raised by A till this instant.
the horizontal as shown in the Figure. The coefficient of
friction is equal to µ. Find the angle β which the thread
Circular Dynamics
must form with the inclined plane for the tension of the
thread to be maximum. What is it equal to? Q.29 An astronaut is rotating in a rotor having
vertical axis and radius 4m. If he can withstand upto

acceleration of 10 g. Then what is the maximum number
m
of permissible revolutions per second? 60o

 Q.30 A racing-car of 1000kg moves round a banked


track at a constant speed of 108 km ms-2. Assuming the
Q.24 A plank of mass with a block of mass m2 placed on total reaction at the wheels is normal to the track and
it lies on a smooth horizontal plane. A horizontal force the horizontal radius of inclination of the track to the
growing with time t as F=at(a is constant) is applied horizontal and the reaction at the wheels.
to the plank. Find how the accelerations of the plank
and of the bar w2 depend on t, if the coefficient of Q.31 A man whirls a stone around his lead on the end
friction between the plank and block is equal to k. Draw of a string 4metre long. If the stone has a mass of 0.4 kg
approximate plots of these dependences. and the string will break if the tension in it exceeds 8 N,
what is the smallest angle the string can make with the
Q.25 A horizontal plane with the coefficient of friction horizontal? What is the speed of the stone? 40o
k supports two bodies: a bar and an electric motor with
a battery on a block. A thread attached to the bar is
wound on the shaft of the electric motor. The distance
4 . 5 6 | Forces and Laws of Motion

Q.32 A boy whirls a stone in a horizontal circle of radius speed of the particle and the inclination of the string to
1.5m and 2m above the ground by means of a string. the vertical at the instant of the motion when the tension
The string breaks and the stone files off horizontally, in the string is equal to the weight of the particle.
striking the ground 10m away. What is the centripetal
acceleration during circular motion? Q.40 A hemispherical bowl of radius R is rotated about
its axis of symmetry which is kept vertical. A small block
Q.33 A stone is fastened to one end of a string and is is kept in the bowl at a position where the radius makes
−2
whirled in a vertical circle of radius R. Find the minimum angle ms with the vertical. The block rotates with the
speed the stone can have at the highest point of the bowl without any shipping. The frictional coefficient
circle. between the block and the bowl is β=54o28’. Find the
range of angular speed for which the block will not slip.
Q.34 A stone of mass 1kg is attached to one end of a
string of length 1m and breaking strength 500N, and Q.41 A table with smooth horizontal surface is fixed in a
is whirled in a horizontal circle on a frictionless table cabin that rotates with a uniform angular velocity m / s2
top. The other end of the string is kept fixed. Find the in a circular path of radius R=700m. A smooth groove
maximum speed the stone can attain without breaking AB of length L=7m is made on the surface of the table.
the string. (k1 + k 2 ) x
The groove makes an angle with the radius OA
m
Q.35 A circular automobile test track has a radius of
of the circle in which the cabin rotates. A small particle
200m. The track is so designed that when a car travels
if kept at the point A in the groove and is released to
at a speed of 100 kilometer per hour, the force between
move along AB. Find the time taken by the particle to
the automobile and the track is normal to the surface of
reach the point B.
track. Find the angle of the bank.

Q.42 A table with smooth horizontal surface is placed


Q.36 A block of mass M is kept on a horizontal ruler.
in a cabin which moves in a circle of a large radius
The friction coefficient between the ruler and block
R. A smooth pulley of small radius is fastened to the
mg2 cos α
is v = . The ruler is fixed at one end and the table. Two masses of m and 2m are placed on the table
2asin2 α connected through a string going over the pulley.
block is at a distance L from the fixed end. The ruler is Initially the masses were at rest. Find the magnitude of
rotated about the fixed end. Find the maximum angular the initial acceleration of the masses as seen from the
speed for which block will slip. cabin and the tension in the string.

Q.37 A motorcycle has to move with a constant speed Q.43 A particle of mass m moves along a horizontal
on an over bridge which is in the form of a circular circle of radius R such that normal acceleration of particle
are of radius R and has a total length L. Suppose the varies with time as Tab=mgtanθ, Tab=mg/cosθ, T’=mgcosθ.
motorcycle starts from the highest point. (a) What can
where K is a constant. Calculate
its maximum velocity be for which contact with road is
not broken at the highest point? (b) If the motorcycle (i) Tangential force on particle at time t
goes at speed grφ2 (r − r φ) =2lgm times the maximum (ii) Total force on particle at time t
found in part (a). Where will it lose the contact with the (iii) Power developed by total force at time t and
road? (c) What maximum uniform speed can it maintain
(iv) Average power developed by total force over first t
on the bridge if it does not lose contact anywhere on
second
the bridge?
Q.44 A smooth sphere of radius R is made to translate in
Q.38 A simple pendulum is suspended from the ceiling
a straight line with a constant acceleration a. A particle
of a car taking a turn of radius 10m at a speed of 36km/h.
kept on the top of the sphere is released from there at
Find the angle made by the string of the pendulum with
zero velocity with respect to the sphere. Find the speed
the vertical if this angle does not change during the
of the particle with respect to the sphere as a function
turn. Take kmh-2.
of the angle Mg a2 + 4h2 it slides.
4h
Q.39 A heavy particle hanging from a fixed point by
Q.45 A uniform circular ring of mass per unit length
a light inextensible string of length ms−2 is projected
  2(Mg− B)
horizontally with speed − ( x + y ) / 2cm / s2 , find the and radius R is rotating with angular velocity
g
P hysi cs | 4.57

mg2 cos α µMg µMg


f (k1 − k 2 )
= about its own axis in a gravity free (B) fA = µMg, fB =, T =
m1 + m2 2 2
space. Find the tension in the ring. (C)
(C) F ≥ (m1 + m2 + m3 )µ g

(D) F ≤ (m1 + m2 + m3 )µ g
Q.46 If a particle is rotating in a circle of radius R with
velocity at an instant v and the tangential acceleration is
a. Find the net acceleration of the particle. Q.4 Blocks A and B in Figure are connected by a bar
of negligible weight. If mass of A and M are 170 kg
each and µ A = 0.2 and µB = 0.4 , where µ A and µB are
Q.47 A metal ring of mass m and radius R is placed on
the coefficients of limiting friction between blocks and
a smooth horizontal table and is set rotating about its
own axis in such a way that each part of the ring moves (
plane, calculate the force in the bar. g = 10 m / s2 )
with speed v. Find the tension in the ring.
B
Q.48 A car goes on a horizontal circular road of
radius R the speed is increasing at a constant rate
A 8
k1 m1 + k 2 m2
cos αmin = a. The friction coefficient is 15
m1 + m2
(A) 150 N (B) 75 N (C) 200 N (D) 250 N
1  1
=α tan−1  −  . Find the speed at which the car will
2  µ Q.5 A person, standing on the floor of an elevator, drops
just skid. a coin. The coin reaches the floor of the elevator in a
time t1 . If the elevator is stationary and in time t2 if it is
Exercise 2 moving uniformly. Then
(A) t1 = t2
Forces and Laws of Motion
(B) t1 < t2
Single Correct Choice Type
(C) t1 > t2

Q.1 A chain of length L and mass M is hanging by fixing (D) t1 > t2 or t1 < t2 depending on whether the
its upper end to rigid support. The tension in the chain lift is going up or down.
at a distance x from the rigid support is

m2g3 cos α Q.6 How large must F be in the Figure shown to given
s=
6a2 sin3 α the 700 gm block an acceleration of 30 cm / s2 ? The
coefficient of friction between all surfaces is 0.15.
Q.2 A block A kept on an inclined surface just begins
to slide if the inclination is 30o. The block is replaced by 200gm
another block B and it is found that it just begins to slide
if the inclination is 40o. 700gm F
(A) Mass of A > mass of B.
(B) Mass of A < mass of B (A) 4 N (B) 2.18 N (C) 3.18 N (D) 6N

(C) Mass of A = mass of B


Q.7 If the force which acting parallel to an inclined plane
(D) All the three are possible. of angle α just sufficient to draw the weight up in n times
the force which will just let it be on the point of sliding
Q.3 The arrangement shown in the Figure, the system down, the coefficient of friction will be
of masses m1 ,m2 and m3 is being pushed by a force
(n− 1) (n+ 1)
F applied on m1 horizontally. In order to prevent the= (A) µ tan α (B) µ
= tan α
downwards slipping of m2 between m1 and m3 . If n+1 n−1
coefficient of friction between m2 and m3 is µ and all (C)
= µ ntan α (D) µ= (n + 1)tan α

the other surfaces are smooth, the minimum value of F
3
(A) fA = µMg, fB = 0, T =0
4
4 . 5 8 | Forces and Laws of Motion

Q.8 Two blocks A and B of masses m and M are placed


A P
in a platform as shown in the Figure. The friction
coefficient between A and B is µ but there is no friction B
between B and the platform. The whole arrangement is C
placed inside an elevator which is coming down with an
acceleration a(a < g). What maximum horizontal force F (A) 90 N (B) 60 N (C) 80 N (D) 100 N
can be applied to A without disturbing the equilibrium
of the system? Q.12 A block of mass rests on a rough horizontal
plane relative to which
A the coefficient of friction
F m is µ. A light string
f attached to the body
B M T passes over a light
pulley and carries at its 45°
(A) 2µmg (B) 2µm(g – a) other end a mass m2 . m2 m1
When the system just
(C) 2µm(g+a) (D) 2µma
begins to move, the
value of µ is
Q.9 A body of mass m1 is placed on a horizontal plank of
m2 m m2 m2
mass m2 which rests on a smooth horizontal table. The (A) (A) 2 (B) (B)
coefficient of friction between the mass m1 and plank 2m1 − m2 2m1 − m2 2m1 + m2 2m1 + m2
is µ. A gradually increasing force F depending on time m2 m2 m2 m2
t as F=at where a is constant is applied to the plank. (C) (C) (D) (D)
2m2 + m1 2m2 + m1 2m2 − m1 2m2 − m1
The time t0 at which the plank starts sliding under the
mass is
m1µg (m1 + m2 )µg m2µg m1m2µg
(A) (B) (C) (D) Multiple Correct Choice Type
a a a a

Q.14 In the arrangement shown in the Figure pulley


Q.10 Block A is placed on block B whose mass is greater
is smooth and massless and string is light. Friction
than that of A. There is friction between blocks while
coefficient between A and B is µ. Friction is absent
the ground is smooth. A horizontal force P increasing
between A and plane. Select the correct alternative(s)
linearly with time begins to act on A. The accelerations
a1 and a2 of A and B respectively are plotted in a graph
against time. Which of the following graphs represents
the real situation? B
a a A
a2 a2
 Fixed
a1 a2 a1
(A) (B) a1
(A) acceleration of the system is zero if
t t m − mA
a a µ≥ B tan θ and mB > mA
a1 a1 2mB
(B) Force of friction between A and B is zero if
a2 a2 a2
(C) (D) a2 mA = mB
a1 a1
t t (C) B moves upwards if mB < mA
(D) Tension in the string is mg ( sin θ − µ cos θ ) if
Q.11 Find the least horizontal force P to start motion m= m= m
A B
of any part of the system of the three blocks resting
upon one another as shown in Figure. The weight of
blocks are A=300 N, B= 100 N, C=200 N. Between A
and B, µ=0.3. Between B and C, µ=0.2. Between C and
the ground µ=0.1.
P hysi cs | 4.59

Q.15 (A, D) Two blocks A and B of Q.18 Assertion: The law of conservation of linear
30°
mass 10 kg and 20 kg respectively momentum, as applied to a single particle, is equivalent
are placed as shown in Figure. to Newton’s first law of motion.
Coefficient of friction between all
Reason: As Newton’s first law states in the absence
the surfaces is 0.2 (g=10 m/s2)
of external force state of motion of a body does not
(A) Tension in the string is 306 N change.
(B) Tension in the string is 132 N A B
(C) Acceleration of block B is 2.6m/s 2 Q.19 Assertion: The impulse of a force can be zero
(D) Acceleration of block B is 2.6m/s 2 even if force is not zero.
Reason: The impulse of a force is equal to change in
Q.16 In the arrangement shown in the Figure. all surface momentum of a body.
are smooth. Select the correct alternative(s)
Q.20 Assertion: If a book is placed on table at rest then
force exerted by table on the book and weight of the
B
A book formation reaction pairs according to Newton 3rd
law of motion
 Fixed Reason: Since both are equal in magnitude and
opposite in directions.
(A) For any value of θ acceleration of A and B are equal
Q.21 Assertion: The mass of a body can be regarded
(B) Contact force between the two blocks is zero if as a quantitative measure of the resistance of a body to
mA / m
=B tan θ acceleration by a given force.
(C) Contact force between the two is zero for any value Reason: The acceleration produced by a given force is
of mA or mB inversely proportional to mass being accelerated.
(D) Normal reactions exerted by the wedge on the block
are equal. Q.22 Assertion: While conserving the linear momentum
of the system we must specify the reference frame.
Q.17 Two blocks A and B of equal mass m are connected Reason: Like velocity, momentum also depends on the
through a massless string and arranged as shown in reference from of observer.
Figure. Friction is absent everywhere. When the system
is released from rest. Comprehension Type
(A) Tension in string is mg/2 C
Paragraph 1: A ball of mass m
(B) Tension in string is mg/4 is connected with the string AB 
(C) Acceleration in string is g/2 and BC respectively as shown in
the figure. Now string AB is cut. A
(D) Acceleration in string is 3g/2 Answer the following questions B

Assertion Reasoning Type Q.23 Tension in the string AB and BC respectively the
string AB is cut
Each of the question given below consists of two
statements, an assertion and reason. Select the number (A) mg cotB, mg cosB (B) mg tanB, mg cosB
corresponding to the appropriate alternative as follows
(C) mg tanB, mg secB (D) mg cotB, mg secB
(A) If both assertion and reason are true and reason is
the correct explanation of assertion Q.24 Tension in the string BC just after the string AB is
(B) If both assertion and reason are true but reason is cut
not the correct explanation of assertion (A) mg sinB (B) mg cosB
(C) If assertion is true but reason is false (C) mg tanB (D)mg secB
(D) If assertion is false but reason is true.
4 . 6 0 | Forces and Laws of Motion

Q.25 If string BC is cut instead of AB, what is the tension


(A) − gtan
(A)θ − gtan θ (B) + gtan
(B)θ + gtan θ
in the string AB just after gtan θ gtan θ gtan θ gtan θ
(C) − (C) − (D) + (D) +
(A) mg cosB (B) mg tanB 2 2 2 2
(C) mg sinB (D) zero
Passage 3: An arrangement designed to measure
the acceleration due to gravity at a place consist of
Q.26 If the whole system is placed in an automobile,
two blocks A and B, of mass m and 2m respectively
what is the acceleration required to be given to it so
connected to each other by means of a light inextensible
that even after cutting the string AB, it remains in the
string passing over a light frictionless pulley as shown in
same position
the Figure. A light and very rough plank, rigidly held
(A) g tanB, right ward (B) g cotB, right ward in position, supports the block A. The system, it is
(C) g tanB, left ward (D) g cotB, left ward observed does not move at all. The portion of the string
OA, is initially horizontal. Assume that the acceleration

Passage 2: A block of mass m slides down a smooth


(
due to gravity, g = 10 m / s2 )
incline of mass M and length l, solely as a result of A
O
m
the force of gravity. The incline is placed on a smooth Clamp
horizontal table as shown in Figure. Let us denote the
Plank
coordinate system relative to the table as S1 and the
coordinate system relative to the incline as Sφ
m 2m B

M
Q.31 The net force due to plank, acting on the block A,
has magnitude
(A) 2mg (B) mg (C) 3mg (D) 5mg
l

Q.27 The acceleration of m in the S’ frame is Q.32 The magnitude of the force exerted on the pulley
by the clamp is, when the system is in equilibrium
(M+ m)gsin θ
(M+ m)gsin (M+ m)gsin θ
(A) (A) 2 (B)θ (B) (M+ m)gsin θ 2 2 mg
(M+ m)gsin
(M θ 2 θθ(M
M++m)gsin + m)gsin
(M
m++m)gsin
2 θ
θMsin2 θθ (A) 4mg (B) 4mg / 3 (C) (D) 2 2mg
(A) M +(A)
msin θmsin (B) m +(B)Msin
3
2 2 2
(M
M−+m)gsin
msin θ
M−+m)gsin
(M θmsin θθ(M
m + m)gsin
Msin
m+θ
(M θ 2 θθ
Msin
m)gsin
(C) (C) 2 (D) (D) Q.33 The plank is suddenly broken by an impulsive force,
(M− m)gsin
(M θ 2 θθ(M
M−+m)gsin M++m)gsin
(M+θm)gsin
msin θ θ
(C) M +(C)
msin θmsin (D) (D) M + msin θ
M + θmsin θ M + msin
M + msin 2 2 M +θmsin θ acting downwards. The instantaneous accelerations of A
and B, just after the plank is removed, are respectively,
Q.28 The acceleration of the incline in the S frame (A) 10 m/s2 and 10 m/s2 (B) 20 m/s2 and 3.33 m/s2

 mgsin θ cos θ   mgsin θ cos θ  (C) 12 m/s2 and 6.66 m/s2 (D) None of the above
(A)   mgsin θ cos  θ (B) −   mgsin θ cos  θ
mgsin
(A) θ cos
M+mgsin
msin 2 θ
θθcos θ   mgsin
(B) − θ cos
M+mgsin
msin 2 θ
θθcos
 θ 
(A) (A) M + msin 2  θ  (B) −(B)
 −  M + msin 2
 θ 

M+M
 Mgsin msin 2
θθ 2 θ  M+ Mmsin 2
θθ 2 θ  Passage 4: A vertical gap 2.2 cm wide of infinite extent
θ
+ cos
msin  Mgsin θ cos
+ msin
(C)   Mgsin θ cos θ  (D) −   Mgsin θ cos θ contains a fluid of viscosity 2.0 NS / m2 and specific
Mgsin
(C) 2 θ
θ cos  θ  (D) Mgsin 2 θ
θ cos
− +Mgsin  θ 
(C) (C)M+Mgsin
msin θθcos
2 (D) −(D) M msin θθcos
 M + msin  θ   − M + msin 2 θ  gravity 0.9. A metallic plate 1m X 1m X 0.2cm, which
 M+M msin 2
θ 2 θ 
+ msin  M+ Mmsin 2
+ msinθ 2 θ 
is in the middle of the gap, is to be lifted up with a
constant speed 0.15 m/sec through the gap. The weight
Q.29 The force exerted by the small m on the wedge of of the plate is 48N. Assuming pulley is massless and
mass M frictionless, string is also massless. (g=10 m/s2)
Mmg Mmg
(A) mgcos θ mgcos θ
(A) (B) (B) 2
M + msinM2 θ+ msin2 θ

mg mg
(C) (C) (D) None(D) None
cos θ cos θ m-83kg

60°
Q.30 At what acceleration ax (in the S frame) must the
0.2cm
incline be accelerated to prevent m from sliding
2.2cm
P hysi cs | 4.61

Q.34 Buoyant force acting on the plate Q.42 Three identical particles are joined together by
(A) 1800 N (B) 900 N (C) 180 N (D) 18 N a thread as shown in figure. All the three particles are
moving on a smooth horizontal plane about point O.
Q.35 Net frictional force exerted by the liquid on the If the velocity of the outermost particle is v 0 , then the
plate ratio of tensions in the three sections of the string is
(A) 30 N (B) 60 N (C) 15 N (D) 120 N (A) 3:5:7 (B) 3:4:5 (C) 7:11:6 (D) 3:5:6

Q.36 Tension in the string Q.43 The kinetic energy k of a particle moving along a
circle of radius R depends on the distance covered s as
(A) 90 N (B) 108 N (C) 240 N (D) 120 N
k= as2 where a is a constant. The total force acting on
the particle is:
Q.37 For doing so the kinetic friction between the  s2 
1/2
s2 R2
inclined plane and the block should be equals to (A) 2a (B) 2as   (C) 2as (D) 2as
R  R2  s2
3 3 1 1  
(A) (B) (C) (D)
4 8 3 2 3
Multiple Correct Choice Type

Q.38 A block of mass m is placed on a plank, which is Q.44 A car of mass M is moving on a horizontal circular
pivoted at one end. The plank is slowly turned as shown path of radius r. At an instant its speed is v and is
in Figure. The friction coefficient between block and increasing at a rate a-
plank is 0.8. Angle between ground and plank friction
(A) The acceleration of the car is towards the centre of
force between block and plank at which the block starts
the path
sliding is
(B) The magnitude of the frictional force on the car is
block greater than mv 2 / R
ground
plank (C) The friction coefficient between the ground and the
car is not less than a/g
(A) 20o (B) 45o (C) 30o (D) 35o
(D) The friction coefficient between the ground and the
car is µ =tan−1 v 2 / Rg

Circular Dynamics Q.45 A circular road of radius r is banked for a speed


of v=40km/h. A car of mass m attempts to go on the
Single Correct Choice Type circular road. The friction coefficient between the tyre
and the road is negligible. Then-
Q.39 Two bodies of mass 10k and 5kg moving in (A) The car cannot make a turn without skidding
concentric orbits of radii R and r such that their periods
(B) If the car turn at a speed less than 40km/h, it will slip
are the same. Then the ratio between their centripetal
down.
acceleration is:
(C) If the car turns at the correct speed of 40km/h the
(A) R/r (B) r/R (C) R2/r2 (D) r2/R2
force by the road on the car is equal to mv 2 / r
Q.40 A string breaks if its tension exceeds 10 newton. A (D) If the car turns at the correct speed of 40 km/h, the
stone of mass 250 mg tied to this string of length 10cm force by the road on the car is greater that mg as well as
is rotated in a horizontal circle. The maximum angular greater than mv2/r
velocity of rotation can be:
Q.46 Figure shows a rod of length L pivoted near an end
(A) 20 rad/s (B) 40 rad/s (C) 100 rad/s (D) 200 rad/s
and which is made to rotate in a horizontal plane with a
constant angular speed.
Q.41 A stone of mass of 16kg is attached to a string 
A ball of mass m is
144m long and is whirled in a horizontal circle. The L
suspended by a string
maximum tension the string can withstand is 16 newton.
also of length L from the L
The maximum speed of revolution of the stone without 
other end of the rod. If θ is T m
breaking it, will be:
the angle made by sting
(A) 20ms-1 (B) 16ms-1 (C) 14ms-1 (D) 12ms-1 with the vertical, then-
4 . 6 2 | Forces and Laws of Motion

(A) T sin θ= mω2L (1 + sin θ ) (B) T cos θ =mg Q.52 Assertion: A cyclist always bends inwards
while negotiating a curve
ω2L (1 + sin θ ) Reason: By bending he lowers his centre of gravity
(C) tan θ = (D) None of above
g
Q.53 Assertion: The tendency of skidding/overturning

Q.47 A person applies a constant force F on a particle is quadrupled, when a cyclist doubles his speed of
of mass m and finds that the particle move in a circle of turning.
radius r with a uniform speed v. v2
(A) This is not possible Reason: tan θ =
rg
(B) There are other forces also on the particle
(C) The resultant of other forces is mv 2 / r towards centre Q.54 Assertion: On a banked curved track, vertical
(D) The resultant of the other forces varies in magnitude component of normal reaction provides the necessary
as well as direction centripetal force.
Reason: Centripetal force is always required for turning.
Assertion Reasoning Type
In each of the following questions, a statement of Comprehension Type Questions
Assertion (A) is given followed by a corresponding
statement of Reason (R) just below it/of the statements, Passage 1: A block of mass m moves on a horizontal
mark the correct answer as circle against the wall of a cylindrical room of radius
(A) If both assertion and reason are true and reason is R. The floor of the room on which the block moves is
the correct explanation of assertion. smooth but the friction coefficient between the block
(B) If both assertion and reason are true but reason is and the side wall is µ . The block is given initial velocity
not the correct explanation of assertion. v o . Then answer the following questions.
(C) If assertion is true but reason is false.
Q.55 What is the tangential acceleration of the block?
(D) If assertion is false but reason is true. 2
(A) µg (B) −µg (C) µv 2 / R (D) −µv / R
(E) If both assertion and reason are false.
Q.56 What is the value of velocity v as the function of
Q.48 Assertion: Centripetal and centrifugal forces time t?
cancel each other
1 1 µt 1 1 µt
Reason: This is because they are always equal and (A) = + (B) = −
opposite. v v 0 2R v v 0 2R

1 1 µt 1 1 µt
Q.49 Assertion: A cyclist bends inwards from his (C) = + (D) = −
v v0 R v v0 R
vertical position, while turning to secure the necessary
centripetal force.
Q.57 What is the value of velocity v as the function of
Reason: Friction between the tyres and road provides
distance x travelled on the circumference?
him the necessary centripetal force. 2µ µ
− − x
(A) v = v o e R (B) v = v o e R
Q.50 Assertion: The tendency of skidding or
overturning is quadrupled, when a cyclist double his  − x

=
(C) v v o 1 − e R  (D) v = v o

speed of turning.  
 

Reason: Angle of bending increases as velocity of Passage 2: In a rotor, a hollow vertical cylindrical
vehicle increases. structure rotates about its axis and a person rests
against the inner wall. At a particular speed of the rotor,
Q.51 Assertion: On banked curved track, vertical the floor below the person is removed and the person
component of normal reaction provide the necessary hangs resting against the wall without any floor. If the
centripetal force. radius of the rotor is 2m and the coefficient of static
friction between the wall and the person is 0.2. Find the
Reason: Centripetal force is always required for motion
following parameters and relations.
in curved path.
P hysi cs | 4.63

Q.58 If v is the velocity of rotation of rotor and N be the Q.62 The bob of a simple pendulum is given a velocity
reaction of wall, then- 10m/s at its lowest point. Mass of the bob is 1kg and
(A) N=mg string length is 1 m.
    Column I Column II
(B) F = F1 + F2 ⇒| F |= 102 + 52 + 2.10.5cos120o = 5 3N
(A) Minimum tension in string (in (p) 50
2 Newton)
 mv 2 
(mg)
2
(C) N
= +  (B) Magnitude of acceleration of (q) 60
 r 
  bob when the string is horizontal
(in m / s2 )
(D) None of these
(C) Minimum magnitude of accelera- (r) zero
Q.59 In order to man remain in equilibrium we must tion of bob (in m / s2 )
have (D) Tangential acceleration at the
R highest point (in m / s2 ) (s) 10 65
fs

Q.63 A car of mass 500kg is moving in a circular road


N
of radius 35 / 3 . Angle of the banking of road is 30.
mg
1
Coefficient of friction between road and tires is µ = .
2 3
Match the following:
(A) µmg =
N (B) f2 = µmg
Column I Column II
(C) µN =mg (D) None of these
(A) Maximum speed (in m/s) of car for
safe turning (p) 5 2
Q.60 The value of velocity will be given by –
(B) Minimum speed (in m/s) of car for (q) 12.50
C safe turning
(C) Speed (in m/s) at which friction
force between tires and road is zero (r) 210
D B
(D) Friction force (in 102 Newton) 350
between tires and road if speed is (s)
3
u 350
A m/s
6
rg
(A) v= µrg (B) v =
µ
Previous Years’ Questions
g µg
(C) v = (D) v =
µr r Forces and Laws of Motion
Q.1 In the Figure, the blocks A, B and C have masses 3
Match the Columns
kg, 4 kg, and 8 kg respectively. The coefficient of sliding
Q.61 A particle is suspended from a string of length ‘R’. friction between any two surfaces is 0.25. A is held at rest
by a massless rigid rod fixed to the wall, while B and C are
It is given a velocity u = 3 Rg . Match the following
connected by a light flexible cord passing around a fixed
Column I Column II frictionless pulley. Find the force F necessary to drag C
along the horizontal surface to the left at a constant
(A) Velocity at B (p) 7 mg speed. Assume that the arrangement shown in the
(B) Velocity at C Figure. i.e. B on C and A on B, is maintained throughout.
(q) 5gR
(Take g=10ms2) (1978)
(C) Tension at B (r) 7gR
(D) Tension at C (s) 4mg A

B
F
C
4 . 6 4 | Forces and Laws of Motion

Q.2 A uniform rope of length L and mass M lying on a acceleration of 0.2 m / s2 . The acceleration due to gravity
smooth table is pulled by a constant force F. What is the is g=9.8 m/s2. (1989)
S
tension in the rope at a distance l from the end where
the force is applied? (1978)

Q.3 A block of mass 2 kg slides on an inclined plane which 2.9 kg


makes an angle of with the horizontal. The coefficient
of friction between the block and the surface is 3 / 2
. What force along the plane should be applied to the
block so that it moves (a) down and (b) up without
any acceleration (Take g = 10 m / s2 ) (1978) 1.9 kg

Q.4 Two blocks connected by a massless string slides


down an inclined plane having an angle of inclination Q.8 A hemispherical bowl of radius R=0.1 m is rotating
37o. The masses of the two blocks are M1=4 kg and about its own axis (which is vertical) with an angular
M2=2 kg respectively and coefficients of friction of M1 velocity ω. A particle of mass 10−2 kg on the frictionless
and M2 with the inclined plane inner surface of the bowl is also rotating with the same
M
are 0.75 and 0.25 respectively. 1 ω. The particle is at height h from the bottom of the
Assuming the string to be bowl.
taut, find (a) the common M (a) Obtain the relation between h and ω. What is the
acceleration of two masses 2
minimum value of ω needed, in order to have a non-
and (b) the tension in the zero value of h?
37°
string. (sin37o=0.6,
(b) It is desired to measure (acceleration due to gravity)
cos37o=0.8) (Take g=9.8 m/s2) (1979)
using the setup by measuring h accurately. Assuming
that R and ω are known precisely and that the least count
Q.5 Masses M1, M2 and M3 are connected by strings in the measurement of h is 10-4 m, what is minimum
of negligible mass which passes over massless and possible error ∆g in the measured value of g?
frictionless pulleys P1 and P2 as shown in Figure. The
(1993)
masses move such that portion of the string between P1
and P2 is parallel to the inclined plane and portion of the Q.9 A smooth semicircular wire track of radius R is fixed
string between P2 and M3 is horizontal. The masses M2 in a vertical plane. One end of a massless spring of
and M3 are 4.0 kg each and coefficient of kinetic friction natural length 3R/4 is attached to the lowest point O of
between the masses and surfaces is 0.25. The inclined the wire track. A small ring of mass
plane makes an angle of 37° with the horizontal. m which can slide on the track is
P (1981) attached to the other end of the
1
spring. The ring is held stationary
at point P such that the spring
M makes an angle with the vertical. C
2
P2 The spring constant k=mg/R. P
Consider the instant when the 60°
M1
M3 ring is making an angle 60o with
37°
the vertical. The spring is released O
(a) Draw the free body diagram of
Q.6 A block of mass m rests on a horizontal floor with the ring. (b) Determine the tangential acceleration of the
which it has a coefficient of static friction µ. It is desired ring and the normal reaction?  (1996)
to make the body move by applying the minimum
possible force F. Find the magnitude of F and direction Q.10 Two blocks of mass m1 = 10 kg and m2 = 5 kg
in which it has to be applied. (1987) connected to each other by a massless inextensible
string of length 0.3 m are placed along a diameter of
Q.7 Two blocks of mass 2.9 kg and 1.9 kg are suspended turn table. The coefficient of friction between the table
from a rigid support S by two inextensible wires each of and m1 is 0.5 while there is no friction between m2 and
length 1 m. The upper wire has negligible mass and the the table. The table is rotating with an angular velocity
lower wire has a uniform mass of 0.2 kg/m. The whole of 10 rad/s about the vertical axis passing through
system of blocks, wires and support have an upward centre O. The masses are placed along the diameter of
P hysi cs | 4.65

the table on either side of center O such that the mass and gravity is neglected, then the time after which the
m1 is at a distance of 0.124 m from O. The masses are bead starts slipping is (2000)
observed to be at rest with respect to an observer on the µ µ 1
turn of table. (1997) (A) α (B)
α
(C)
µα
(D) Infinitesimal
(A) Calculate the frictional force on m1 .
(B) What should be the minimum angular speed of the Q 14. A small block is shot into each of the four track
turn table, so that the masses will slip from this position? as shown below. Each of the track rises to the same
(C) How should the masses be placed with the string height. The speed with which the block enters the
remaining taut so that there is no frictional force acting track is the same in all cases. At the highest point of
on the mass m2 ? the track, the normal reaction is maximum in (2001)

Q.11 Block A of mass m and block B of mass 2m are


placed on a fixed triangular wedge by means of a
massless, inextensible string and a frictionless pulley v v
as shown in Figure. The wedge is inclined at 45o to the (A) (B)
horizontal on both the sides. The coefficient of friction
between block A and wedge is 2/3 and that between
block B and the wedge is 1/3. If the blocks A and B
v v
released from rest, find (A) the acceleration A,
(C) (D)
(B) Tension in the string and
(C) The magnitude and direction of the force of friction
acting on A (1997) Q.15 A simple pendulum of length L and mass (bob)
M is oscillating in a plane about a vertical line between
angular limits −φ and + φ . For an angular displacement
A B ( )
θ θ < φ , the tension in the string and the velocity of
m 45° 45° 2m the bob are T and v respectively. The following relations
hold good under the above conditions. (1986)
Q.12 Two blocks A and B of equal masses are released (A) T cos θ =Mg
from an inclined plane of inclination 45o at t=0. Both Mv 2
the blocks are initially at rest. The coefficient of kinetic (B) T − Mgcos θ = L
friction between the block A and inclined plane is 0.2 (C) The magnitude of the tangential acceleration of the
while it is 0.3 for block B. Initially the block A is 2 m bob α = gsin θ
T
behind the block B. When and where their front faces
will come in a line?  (2004) =(D) T Mgcos θ

2m
A Q.16 A reference frame attached to the earth (1986)
B (A) Is an inertial frame by definition
(B) Cannot be an inertial frame because the earth is
revolving round the sun

B (C) Is an inertial frame because Newton’s laws are


45°
A applicable in this frame
(D) Cannot be an inertial frame because the earth is
rotating about its own axis
Circular Dynamics
Q.17 A point mass of 1 kg collides elastically with a
Q 13. A long horizontal rod has a bead which can slide stationary point mass of 5 kg. After their collision, the 1
along its length and is initially placed at a distance L from kg mass reverses its direction and moves with a speed of
one end A of the rod. The rod is set in angular motion 2ms-1. Which of the following statement(s) is/are correct
about A with a constant angular acceleration α .If the for the system of these two masses. (2010)
coefficient of friction between the rod and bead is µ , (A) Total momentum of the system is 3kg−ms −1
4 . 6 6 | Forces and Laws of Motion

(B) Momentum of 5 kg mass after collision is 4kg−ms −1 (C) Radially outwards initially and radially inwards later.
(C) Kinetic energy of the centre of mass is 0.75 J (D) Radially inwards initially and radially outwards later.
(D) Total kinetic energy of the system is 4J
Q.21 A block of mass m1 = 1 kg another mass m2 = 2kg,
are placed together (see figure) on an inclined plane
Q.18 A smooth semicircular wire track of radius R is
with angle of inclination θ. Various values of θ are given
fixed in a vertical plane (Figure). One end of a massless
in List I. The coefficient of friction between the block m1
spring of natural length 3R/4 is attached to the lowest
and the plane is always zero. The coefficient of static
point O of the wire track. A small ring of mass m which
and dynamic friction between the block m2 and the
can slide on the track is attached to the other end of the
plane are equal to µ = 0.3. In
spring. The ring is held stationery at point P such that
List II expressions for the
the spring makes an angle 60o
friction on the block m2 are
with the vertical. The spring
given. Match the correct
constant k=mg/R. Consider the m1
expression of the friction in List
instant when the ring is making m2
II with the angles given in List I,
an angle 60o with the vertical. The
and choose the correct option.
spring is released (a) Draw the C
P The acceleration due to gravity 
free body diagram of the ring. (b)
Determine the tangential 60° is denoted by g.
acceleration of the ring and the O [Useful information: tan (5.5°) ≈ 0.1; tan (11.5°) ≈ 0.2;
normal reaction. (1996) tan (16.5°) ≈ 0.3] (2014)

Q. 19 A small block of mass of 0.1 kg lies on a fixed List I List II


inclined plane PQ which makes an angle q with the
(P) θ = 5° (1) m2g sin θ
horizontal. A horizontal force of 1 N acts on the block
through its centre of mass as shown in the figure. The (Q θ = 10° (2) (m1+m2) g sin θ
block remains stationary if (take g = 10 m/s2) (2012) (R) θ = 15° (3) mm2g cosθ
Q (S) θ = 20° (4) µ(m1 + m2)g cosθ

1N Code:
(A) (P)→(1), (Q)→(1), (R)→(1), (S)→(1)

O P (B) (P)→(2), (Q)→(2), (R)→(2), (S)→(3)
(A) θ = 45° (C) (P)→(2), (Q)→(2), (R)→(2), (S)→(4)
(B) θ > 45° and a frictional force acts on the block (D) (P)→(2), (Q)→(2), (R)→(3), (S)→(3)
towards P.
(C) θ > 45° and a frictional force acts on the block Q.22 The net reaction of the disc on the block is
towards Q. (2016)
(D) θ < 45° and a frictional force acts on the block (A) − m ω R cos ωtjˆ − mgkˆ
2

towards Q.
1
A (B) m ω2R(e2ω t − e−2ω t )ˆj − mgkˆ
2
Q.20 A wire, which passes through
the hole in a small bead, is bent in (C) m ω2R sin ωtjˆ − mgkˆ
the form of quarter of a circle. The 1
(D) m ω2R(e2ω t − e−ω t )ˆj − mgkˆ
wire is fixed vertically on ground as
o
90 B 2
shown in the figure. The bead is
released from near the top of the
wire and it slides along the wire
without friction. As the bead moves from A to B, the
force it applies on the wire is  (2014)
(A) Always radially outwards.
(B) Always radially inwards.
P hysi cs | 4.67

MASTERJEE Essential Questions

JEE Main/Boards JEE Advanced/Boards

Exercise 1 Exercise 1
Q. 18 Q.21 Q.26 Q.1 Q.4 Q.13 Q.17
Q.27 Q.28 Q.20 Q.29 Q.38 Q.43

Exercise 2 Exercise 2
Q.5 Q.14 Q.18 Q.3 Q.6 Q.8 Q.11
Q.14 Q.15 Q.16 Q.17
Previous Years’ Questions
Q.44 Q.45
Q.51 Q.59
Previous Years’ Questions
Q.12 Q.14 Q.17

Answer Key

JEE Main/Boards Q.19 750 N/m

Q.20 1 cm
Exercise 1
Q.21 9 m/s.
Forces and Laws of Motion

Q.22 r (iˆ 12500 + ˆj 6250)m
=
Q.4 100 Ns
Q.23 2.5 × 10 4 km
Q.5 12.5 N 2
Q.24 grφ (r − r φ) =2lgm
Q.14 0.872 m/s, 1.744 m

Q.15 30N
Circular Dynamics
−2 Q.25 0.03 m/s2
Q.16 1.4 ms , 33.6 N
Q.26 0.86 ms−2
Q.17 1000 N, 750 N
 
Q.27 − ( x + y ) / 2cm / s
2
Q.18 40°
−1  10 
Q.28 1.22 m/s2; β =tan  
 7 
4 . 6 8 | Forces and Laws of Motion

Exercise 2
Forces and Laws of Motion
Single Correct Choice Type

Q.1 A Q.2 B Q.3 C Q.4 C Q.5 A Q.6 A Q.7 C

Q.8 B Q.9 B Q.10 A Q.11 C Q.12 B Q.13 A Q.14 A

Q.15 A Q.16 D Q.17 C Q.18 C Q.19 D Q.20 B Q.21 B

Circular Dynamics
Single Correct Choice Type

Q.22 D Q.23 C Q.24 D Q.25 A Q.26 C Q.27 B Q.28 A

Q.29 A Q.30 B Q.31 D Q.32 B Q.33 D

Previous Years’ Questions


Forces and Laws of Motion
Q.1 C Q.2 A Q.3 C Q.4 A Q.5 A Q.6 A Q.7 A

Q.8 D Q.9 C Q.10 A Q.11 D Q.12 A Q.13 B Q.14 A


Q.15 B, D

Circular Dynamics
Q.16 C Q.17 D Q.18 C Q.19 C Q.20 B Q.21 B


JEE Advanced/Boards
Exercise 1
Forces and Laws of Motion

Q.1 (a) 70 kg, (b) 35 kg, (c) 105 kg, (d) zero

Q.2 (a) 750 N, (b) 250 N, Mode (b) should not be adopted

Q.3 (a) T=640 N, (b) T=240 N, (c) T=400 N, (d) T=0, Rope will break in case (a).

Q.4 15 m

Q.5 (a) 7500 N downwards, (b) 32500 N downwards, (c) 32500 N upward

Q.6 (a) accelerated with acceleration 0.5 m / s2 , (b) at rest.

(k1 + k 2 ) x
Q.7
m
Q.8 g/3, 2g/3

Q.9 Tab
= mgtan θ, Tob
= mg / cos θ , =
T' mgcos θ
P hysi cs | 4.69

mg 2
Q.10 a + 4h2 acceleration w1
4h Q.24

Q.11 4N kg w2
2(Mg− B)
Q.12 g
t0 time
mg2 cos α
Q.13 (a) =
f (k1 − k 2 ) ,
m1 + m2
µ(m1 + m2 )g
When t ≤ t0 (where t0 = ) w1 =
w2 =
kg
k1 m1 + k 2 m2 a
(b) cos αmin = t > t0 w1 = at / m1 − km2g / m1 , w2 = k g
m1 + m2

1  1 2l
=
Q.14 α tan−1  −  Q.25 t =
2  µ (3 w + kg)

mg2 cos α m2g3 cos α


Q.15 (a) v = , (b) s = Q.27 aA 2.34
= = m / s2 , aB 1.558m
= / s2 , T 81.8N
2
2asin α 6a2 sin3 α
5
Q.28=
(a) t A 1sec,tB
= 2sec, (b) v A = 5m / s (c) m
3 3
Q.16 (a) fA = µMg, fB =0, T =
0
4
µMg µMg
(b) fA =
µMg, fB =, T = Circular Dynamics
2 2
13µ 5
Q.29 fmax = rev / sec
Q.17 2π
16(2 − 3µ )

g(η − sin α − cos α ) Q.30 45°, 2 × 10 4 N


Q.18 w2
= = 0.5g
η+1
Q.31 θ= 30°, v= 7.7m / s
3 Mgsin θ
Q.19 aA = aC = gsin θ,aB = gsin θ, T =
4 8 Q.32 163.3m / s2
(M+ m)gsin2 α
Q.20 f = Q.33 Rg
M + msin2 α
Q.34 22.36m / s
Q.21 Acceleration of block A is g/8 in horizontal direction
and 5g/8 in vertical direction. Acceleration of Q.35 21°29'
block B is g/2 leftwards. Acceleration of block
C is g/8 rightwards µg
Q.36
L
g g
Q.22 aA
= =
2
,aB
1 + η cot α tan α + η cot α Q.37 (a) Rg
πR
mg(sin α + µ cos α ) (b) a distance along the bridge from the
Q.23 tanB =
µ , Tmin = highest point 3
1 + µ2
(c) gR cos (L / 2R )

Q.38 45°

g
Q.39 v =
3
4 . 7 0 | Forces and Laws of Motion

1/2 1/2 1/2


 g ( sin θ − µ cos θ )   g ( sin θ + µ cos θ )  Q.44 2R ( asin θ + g − gcos θ ) 
Q.40   to    
 R sin θ ( cos θ + µ sin θ )   R sin θ ( cos θ − µ sin θ ) 
   
2 2
Q.45 λR ω
2L
Q.41 2
2
ω R cos θ  v2 
2
Q.46 a + 
R 
 
ω2R 4
Q.42 , mω2R
3 3 mv 2
Q.47
2πR
Q.43 (i) m KR (ii) m K R + Kt 4 ( )
( )
1/ 4
1 Q.48  µ2g2 − a2 R 2 
(iii)mKRt (iv) mKRt  
2

Exercise 2
Forces and Laws of Motion

Single Correct Choice Type

Q.1 C Q.2 A Q.3 A Q.4 A Q.5 A Q.6 B Q.7 A


Q.8 B Q.9 B Q.10 C Q.11 B Q.12 A

Multiple Correct Choice Type

Q.14 A, B Q.15 A, D Q.16 A, C Q.17 B, D

Assertion Reasoning Type

Q.18 A Q.19 D Q.20 D Q.21 A Q.22 A

Comprehension Type

Q.23 C Q.24 B Q.25 D Q.26 A Q.27 A Q.28 B Q.29 B


Q.30 B Q.31 D Q.32 D Q.33 C Q.34 D Q.35 B Q.36 A
Q.37 A

Match the Columns


Q38 A → q; B → r; C → r; D → q

Circular Dynamics

Single Correct Choice Type

Q.39 A Q.40 A Q.41 D Q.42 D Q.43 B


P hysi cs | 4.71

Multiple Correct Choice Type

Q.44 B, C Q.45 B, D Q.46 A, B, C Q.47 B, D

Assertion Reasoning Type


Q.48 D Q.49 C Q.50 B Q.51 E Q.52 B Q.53 A Q.54 D

Comprehension Type
Q.55 D Q.56 C Q.57 B Q.58 B Q.59 C Q.60 B

Match the Columns


Q.61 A → r; B → q; C → p; D → s Q.62 A → p; B → s; C → q; D → r
Q.63 A → r; B → p; C → s; D → q

Previous Years Questions

Forces and Laws of Motion


Q.1 37.5 N Q.2 F(1-l/L) Q.3 a=11.21 N, b=31.21N

Q.4 a= 1.5m / s2 , T=5.2 N Q.5 (a) 4.2 kg, (b) 9.8 N Q.6 mgsinθ

Q.7 (a) 20N, (b) 50N Q.8 (a) 9.89 rad/sec, (b) 9.8 × 10−3 m / s2

Q.9 (a) mg/4,


= (b) atan 5=
3g / 8,N 3mg / 8

Q.10 (a) f=36N inwards, (b) 11.67 rad/sec, (c) m2 at 0.2m and m1 at 0.1 m from O

Q.11 (a) 0, (b) T= 2 2mg / 3 (c) f= mg / 3 2 (down the plane)

Q.12 After A travel a distance of 8 2 m down the plane

Circular Dynamics
5 3 3mg
Q.13 A Q.14 A Q.15 B, C Q.16 B, D Q.17 A, C Q.18 B g,
8 8
Q.19 A, C Q.20 D Q.21 D Q.22 C
4 . 7 2 | Forces and Laws of Motion

Solutions

JEE Main/Boards ∆v 20 – 10 10
In case (i) = = = 2 m/s
∆t 5 5
Exercise 1 ∴F = ma ⇒ 5 = m(2)  … (i)

Forces and Laws of Motion Now further, we want this DV in in 2 s.


Sol 1: A body will preserve its velocity and direction as ∆V 20 – 10 10
anew = ∆t = = = 5 m/s2
long as no force acts on it in its motion. Inertia is in fact new 2 2
the resistance of any physical object to any change in
its motion. ∴Fnew = m(5)  … (ii)

Dividing equation (i) by (ii)


Sol 2: Forces are balanced
m(2)
5
⇒ =
Fnew m(5)
2
a = 0 m/s
25
Fnew = N
2
Objects at rest Objects in motion ⇒Fnew ≡ 12.5 N.
(v = 0 m/s) (v  0 m/s)
Sol 6: Conceptual. Refer to the reading manual.

Stay at rest Stay in motion


Sol 7: 1. Linear inertia: In an isolated system, a body at
(same speed and direction)
rest will remain at rest and a body moving with constant
velocity will continue to do so, unless disturbed by an
Now
external force.
Forces are Unbalanced
2. Gyroscopic Inertia: A body that is set spinning has a
tendency to keep spinning in its original orientation if
There is an acceleration no external force is applied.
3. Rotational Inertia: An object resists any change in its
Acceleration depends Acceleration depends state of rotation. If no external force is applied.
on the ‘net force’ Inversely upon the
mass of object Sol 8: Conceptual, Refer to reading manual.

Sol 9: Absolute unit of weight is Newton (N)


Sol 3: While taking a catch, a cricket player moves his
hands backwards. He has to apply retarding force to Gravitational unit is kg-weight.
stop the moving ball in his hands. If he catches the ball
1 N = 9. 8 kg. wt
abruptly, then he has to apply a large retarding force →

for a short time. So he gets hurt. On the other hand if →
dp → dv
he moves his hands backwards then the player applies Sol 10: F ∝ ; F ∝ m
dt dt
force for longer time to bring the ball at rest. In this case →
he has to apply less retarding force. F = Kma, K= 1

∴F = m a
Sol 4: Dp = FDt
Consequences
Dp = 100 . 1 Ns
1. No force is required to move a body uniformly in a
Dp = 100 Ns. straight line.

v f – vi ∆v 2. Accelerated motion is always due to an external force.


Sol 5: F = ma and a = ⇒
t ∆t
P hysi cs | 4.73

Sol 11: Impulse is defined as the product of the average ∴ N ≡ w = m (g – a)


force and change in time.
t2
Hence weight decreases.

J = Favg (t2 – t1) ; J = ∫ F dt Sol 14: writing down the equations of motion
t1
t2
dp dp
F=
dt
; J= ∫ dt dt
t1
p2

J= ∫ dp ; J = P2 – P1 = DP.
p1
T
T1 T1
Sol 12: Every action has an equal and opposite reaction. m2 11

Example (1) 11.5 m1


N Reaction
force
m1g – T = m1a  … (i)
action

mg T

m1 a
Sol 13: Lift moving uniformly

m1 g
a=0
And for second body
m T – m2g = m2a  … (ii)

Then N – mg = 0 T
∴ N ≡ w = mg.
m2 a
Lift acceleration upward
m2 g
a
Adding (i) and (ii)
m (m1 – m2)g = (m1 + m2)a
 m – m2 
a =  1  g
N = mg + ma
m  m1 + m2 
ma(pseudo force) Here m1 = 11.5 kg, m2 = 11 kg, g = 10 m/s2
mg
Now m1 will descend down by height ‘h’ and m2 moves
N ≡ w = m(g + a) up by the same height h;
1
∴ weight Increases H = ut + at2
2
Lift accelerating downwards: 1
⇒ h = 0. t + × (0. 2) (4)2 = 1. 6 m.
2
ma
And for velocity
N
a N + ma = mg v = u + at
m
m v = 0 + (0. 2) (4)
N = mg – ma
mg v = 0. 8 m/s.
4 . 7 4 | Forces and Laws of Motion

T – m2g = m2a … (ii)

Sol 15: 10 31.5 N (i) + (ii) ⇒ (m1 – m2) g = (m1 + m2)a


0.5
 m – m2   4 −3 10
a =  1  g =  = = 1.4 ms−2
 10
 m1 + m2   4 + 3  7
Let us say the whole system moves forward with an
acceleration ‘a’. And using this value, find the value of T in equation (i)
or equation (ii)
 31.5 
Then a =  10 + 0.5  m/s m1g – m1a = T ⇒ T = m1 (g – a)
 
now put m1 = 4 kg m2 = 3kg
a = 3 m/s2
to get the numerical, after putting values of m1, m2 and
Now let us consider the string. a ⇒ T = m1(g − a) = 33.6 N
a
N 31.5 N T T
m = 0.5 kg Sol 17: m2 m1 F

Now, 31.5 – N = ma
⇒ 31.5 – ma = N The total external Horizontal force applied on the
system is F.
N = 31.5 – (0. 5) (3) F
∴ Acceleration ‘a’ of the system= m/s2
N = 30 Newton. m1 + m2
Given a = 5 m/s2
F
Sol 16: Constraint Equation: ∴5 = m/s2 ∴F = 200 ×5 N
50 + 150

F = 1000 N

Now for finding the tension;

T Consider m2
T1 T N
3kg = m2
m2 a m2 T
m1 = 4 kg (pseudo force)
m2 g
am + am = 0. [ length of string is constant]
1 2
T – m 2a = 0
Let us say m1 moves down with an acceleration ‘a’, then
m2 will move up by an acceleration ‘a’. ∵ T = m2a.
T = 150 × 5
T T = 750 N.
m1 a
Sol 18:

m1 g Tcos θ
T1 θ
50 N
m1g – T = m1a  … (i) T1 sin θ P

T2
T

m2 a 60 N

m2 g
P hysi cs | 4.75

At point P In the whole process, linear momentum along the


x-direction is conserved.
For equilibrium;
∴Initial momentum = 103 × 32 + 8 × 103 × 4
T1 sin θ = 50 … (i) ^
Pi = 64 × 103 kg m/s ( i )
T1 cos θ = T2 … (ii)
Now in the final state
And for the mass; ^
Momentum of car=103 × (–8)= – 8 × 103 ( i )
T2 = 60 N … (iii)
50 Momentum of truck = 8 × 103 ( viˆ )
From (i) and (ii) tan θ = ^
50 T2 = 8v × 103 i
tan θ = ^
60 Pfinal = (– 8 + 8v) × 103 ( i )
θ = tan–1(5/6) = 40º
Pinitial = pfinal
Sol 19: F = kx. ⇒ 64 × 103 = (–8 + 8v) × 103
x = 20 cm = 0. 2 m 64 + 8
∴ v= m/s; v = 9 m/sec
8
150 = k (0. 2)
150 15 → →
k= = × 102 N/m = 7. 5 × 102 N/m. Sol 22: F = m a
0.2 2 →

F
a = ; m = 10g = 10 × 10–3 kg = 10–2 kg
m
Sol 20: T – mg = ma →
(10iˆ + 5ˆj)
T ∴a =
10 –2
m a →
a 103 ˆi + 5 × 102 ˆj
=
Now for amax, we have Tmax


1→ 2
Tmax – mg = ma r = ut+ at
2
Tmax = m (g + a) N = m (9. 8 + 1. 2) N = 2000 (11) → → 1→ 2
Since u = 0, r = at
2
= 22 × 103 N
Tmax →
103 × 25 500 × 25 ˆ
u = î + j
Now Tmax = (Breaking stress) Area 2 2

∴ 22 × 103 = (2. 8 × 108) (pR2) Sol 23: This is just an energy conservation problem on
surface of earth;
22 × 103
R= 1 Gm
28 × 107 π Ei = mv 20 + Ui ; Ui = –
2 R
R= 25 × 10 –6 m 1 Gm
∴Ei = mv 20 –
R = 5 × 10 m –3 2 R

Diameter = 2R ≡ 10 × 10–3 m ≡ 10–2 m. Now finally;


V=0
Sol 21: Before collision  Gm 
Ef = 0 +  –   … (i)
32m/s 4 m/s  R +h
1000 8000
car Truck And Ef = Ei
1 Gm Gm
After collision, ∴ mv 20 – =–
y 2 R R +h
1000 8000 u v 20 G –G
8 m/s x ⇒ – =
2 R R +h
4 . 7 6 | Forces and Laws of Motion

1 1 v
2
r – r' Gm
⇒ = – 0 ∴ =
R + h R 2G 2 (r ')2 g

1 Solving for r’, We get the value of r’.


⇒R+h=
 1 v2 
 – 0
 R 2G  Circular Motion
 
Sol 25:
1
h= –R
 1 v2  w
 – 0
 R 2G 
 

G = 6. 67 × 10–11 m3kg−1s−2 A
R = 64 × 10 m. 5

v0 = 104 m/s
After putting above values we get, h = 2.5 × 104 km

Sol 24: m1 = m2 = m Earth completes 1 rotation in 1 day


rotation
i. e. , ω = 1.
day
θ θ 2π
ω = 1. rad/s
T T 24 × 60 × 60
F π
m2 ω= × 10–2 rad/s
m1 mg 432
and now acceleration at point A;
FBD of m1;
T cos θ a = rω2
r = 6400 km = 6400 × 103 m; r = 64 × 105 m
Gm2
T sin θ F= π2
(r')2 ∴ a = 64 × 105 × × 10–4 m/s2
(432)2
mg a = 0. 03 m/s2

T sin θ = F  … (i)
5
Sol 26: v = 27 km/h = 27 × m/s
T cos θ = mg  … (ii) 18
Gm2 15
(i)/(ii) ⇒ Tan θ = v= m/s
(r ')2 mg 2
Gm

v2 (15)2
tan θ =  … (iii) ar = = = 0. 7
(r ')2 g R 4 × 80

1
r – r' a t = 0. 5 m/s2 = m/s2
tan θ = 2
2 → → →
a net = a r + a t = (0.7)2 + (0.5)2

a net = 0. 86 m/s2
θ

r – r1
2
P hysi cs | 4.77

Sol 27: y Sol 28: B



Q
A
→ O
π/4 a →

x ar
O P →
at
C

A
At point the acceleration will be centripetal acceleration
which is radially directed towards point O. i.e. Let us say the circular turn is of the shape AB.

v 2 ê Now at the starting point of the track i. e. C;
Physically: a = (– r )
r → → →
a = ar + a t
Remember êr and êt are the  êt
unit vectors along radial and
→ v2
êr ar = centripetal acceleration = (– êr )
tangential directions respectively. A R
5
Refer to the figure. O v = 36 km/h = 36 × m/s = 10 m/s
18
v 2 ê

So in this case also a A = (– r ) R = 140 m
r
Now, since the point is in between the points P and Q, → (10)2 5
ar = = m/s2 (– êr )
y 140 7
dv
and given that = 1 m/s
dt
x → dv →
π/4 ∴ at = ( ê ); a t = 1 m/s2( êt )
dt t
→ → →
Now a = a r + a t
– êr

  π a = (0. 7 (– êr )) + 1 êt ) m/s2
angle between OA and OP will be
4
|a| = (0.7)2 + 1 = 0.49 + 1 = 1.49 m/s2 = 1. 22 m/s2
Now let us resolve (– êr ) into î and ˆj .
 1   10 
π π and tan β =   ⇒ β =tan–1  
(– î ) + |– êr | sin (– ˆj )
(– êr ) = |– êr |. cos  0.7   7 
4 4
But since êr and êt are unit vectors
Exercise 2
| êr | = | êt | = 1
1 1 ˆ 1
 (– êr ) = – î – j = ( î + ˆj ) Forces and Laws of Motion
2 2 2
→ Single Correct Choice Type
v2  1 ˆ ˆ 
Now a A = − (i + j) 
r  2  Sol 1: (A) At point A;

v2 ˆ ˆ
aA = – (i + j)
r 2
→ (1)
Put v = 2 cm/s and r = 4 cm, to find a A .

After putting above values we get, aA =− ( xˆ + yˆ ) / 2 cm / s2 T1 A
T1=T2
T2 (2)

m = 10 kg
4 . 7 8 | Forces and Laws of Motion

At point B; T1 Using (i) and (iii) here,


mg=T2
0 + (mg)2 ≤ F ≤ (mg)2 + (µmg)2
mg T =10g ≡ T
2 1
mg ≤ F ≤ mg 1 + µ2
∴ Both the spring show a reading of 10 kg
Sol 4: (C) Tension will always act along the length of the
Sol 2: (B) Here acceleration of the lift is 12 m/s which
2 string and opposing the applied force.
is greater than ‘g’. In option B,

F
2
12m/s

at t = 0 Tension has to act opposite to the applied force, but


there is no string after the end point. Hence the string
The body will undergo a free fall condition. Actually the collapses.
body loses the contact with the floor of the lift.
1 1 In option C, F
∴ s = g t2 = × 10 (0. 2)2 m T
2 2

S = 20 cm.
The tension in the string acts towards the body, thus
Sol 3: (C) Here we need to understand the concept of making the string tough. Hence this is the correct
friction representation.
N
Sol 5: (A) Initially; F is upward Buyoant force
F1
a
f
mg
Mg – F = Ma
We are given that the body is not moving. Hence mg
balancing the forces in both the directions; Mg – Ma = F  … (i)

N – mg = 0  … (i) Now when the mass ‘m’ is released,

F1 – f = 0  … (ii) Balloon starts rising upwards with an acceleration ‘a’.

⇒ N = mg and f = F1. F
Now we don’t know anything about F1.
a
But we know that the force F1 must be less than maximum
static friction i.e. mmg for the body to be at rest.
∴ f = F1 ≤ mmg. And minimum F1 can be zero. (M–m)a
∴ 0 ≤ f ≤ µ mg  … (iii) F – (M – m)g = (M – m)a  … (ii)
Now we know that contact force on the body is Solving (i) and (ii); we get
F= 2
N +f 2
 2a 
m=  M
a+ g
N
Sol 6: (A) Let us assume that the string makes an angle
of ‘θ1’ with the normal of the plane.

f
P hysi cs | 4.79

B→a
m'
θ

a
θ m
A
The only external force acting on the sphere is ‘mg’ FBD of B;
which is vertically downward. Hence the string also
becomes vertical so as to balance the force mg. 
N N
N – m'g = 0 …(ii)
N – m'g = 0 …(ii)
T ∴ T ∴
Sol 7: (C) F. B. D of (1) T – m'a = 0 T – m'a = 0
…(iii) …(iii)
T  ... (i) m’g m’g
T–mg=0
m a a
⇒ T = mg …(i)

mg Using (i) and (iii)


mg = ma + m’a [(i) + (iii)]

 m 
2T cosθ a=  g
 m + m' 
T θ θ θ
T T T T T
m m m 2 Sol 10: (A) Now, the force required to just start the
m 2
motion would be the static friction (fs)
F
(1) (2) (3) f
∴ F = fs = ms mg
2T cos θ – m 2g = 0  … (ii)
i.e. after this point the body starts moving.
From (i) and (ii): 2 (mg) cos θ = m 2g
When the body is moving, kinetic friction acts on the
1 π body (i. e μk mg)
Cos θ = ⇒θ=
2 4 FBD of the body;
F F – fk = ma
Sol 8: (B) The reason for small steps is that the lateral fk
forces are decreased. Imagine taking a large step on → a ms mg – mk mg = ma
concrete. When you put your foot down well in front of
you, it will be pushing forwards on the concrete. And at ⇒ (ms – mk) mg = ma ⇒ a = (ms – mk) g
the end of that step, when that foot is well behind you,
it will be pushing backward on the concrete. The larger a = 0. 98 m/s 2

the step, the larger there forward and backward forces.


Sol 11: (C) Newton’s second law states that the net
Our shoes on Ice can only provide or sustain small force on an object is equal to the rate of change of its
forward/backward forces, before they slip. Hence we try linear momentum.
to reduce the friction. → →
dp
→ d(m v )
⇒F = =
Sol 9: (B) FBD of A; dt dt
if m is constant, then
T →

dv
a mg –T= ma  …(i) ... (i) ≡ m. ≡ ma
dt
mg
4 . 8 0 | Forces and Laws of Motion

Sol 12: (B) FBD of the body; Sol 14: (A)


f
F (Net Air Resistance); θ
N
u(speed) a
mg

mg + F = ma Mg cos α
Mg sin α mg
F
a=g+ ; which is downwards. (i. e opposite) to the
m Given that insect moves very lowly;
direction of displacement (till it reaches maximum ∴ V = 0; Acceleration of the body is also zero.
height)
f = Mg cos α
Since mA > mB; aA < aB
N = Mg sin α
i.e. Body ‘A’ has less downward acceleration when
composed to Body ‘B’. Hence A will go higher than B. Now for the maximum case;
f = fs = mN.
Sol 13: (A) Let ‘x’ be the maximum length that can hang
∴ mN = Mg cos α
hand from the table.
µ (Mg sin α) = Mg cos α
Now say fs be the static friction
1
tan α = ⇒ tan α = 3
L–x µ
But we want to express in terms of θ;

x α + θ = 90o, → α = 90o – θ
tan α = tan (90o – θ)
3 = cotq
M
fs =   . x. g  … (i)
L Sol 15: (A) T θ
T
[ Condition for Equilibrium]
And also we know that fs = mN. w
M
N = (L – x)g When the bird alights on the wire; the wire makes a
L
curve of small angle.
µM
fs = (L – x) g  … (ii) 2 T sin θ = w
L
from (i) and (ii) W
sin θ =  
 2T 
Mx M
g = µ (L – x) g
L L we know that sin θ ≤ 1
x  µ  W  W
⇒ < 1 ⇒T > 
⇒ =   2T  2 
L 1 + µ 

x   µ  1/4 Sol 16: (D) Now Balancing the forces parallel and
 × 100  =  × 100  = × 100 = 20%
L   1 + µ  5/4 perpendicular to the incline surface;
f = mg sin θ
N = mg cos θ
And Net force by surface = f 2 + N2

= (mgsin θ)2 + (mgcos θ)2 = mg = 30 N.


P hysi cs | 4.81

Sol 17: (C) While descending down; (ii) when the body just starts sliding and slides down
The fireman tries to pull the rope down and so there will For case I;
be a tension ‘T’ upwards. N
T F
mg – T = ma; mg – ma = T
2mg mg a
Now given Tmax = Mg sin θ
3 mg cos θ
mg
2mg θ
∴ amin = mg – m ∴ amin = g/3
3
As long as body doesn’t slide;
Sol 18: (C) F = mg sin θ;

40 sin 30º N = mg cos θ

40 ∴ F= f 2 + N2 = mg
50 sin 30º
50 ∴ It remains constant till a particular ‘θ’.
30º 30º For case II;
50 cos 30º
w=5 When the body is sliding down,
f = mN
30
N = mg cos θ
A  2 
50 ∴F = (µN)2 + N2 = N  µ + 1 
 
30N
 2 
C = mg cos θ  µ + 1 
w  
40 As θ increases; cos θ decreases.
B
Hence F decreases.

Fnet = 90 sin 30º – (30 + 5) = 45 – 35


Sol 21: (B) (l–n)
Fnet = 10 N upwards

Sol 19: (D)


2F nL
F
F = 250
F
2F F
m
λ (mass per unit length) =  
L m
θ = 15º Now mass of the part which is hanging= (nL)   = nm
L
And mass of the part which is on the table = (1 – n)m

3F – mg sin α = ma Now total downward force = (nm) g ≡ nmg.


This force has to be balanced by the frictional force
a =  3F – gsin α 
 
m  which is mN ≡ µ [(1 – n) mg]
250 × 3 ∴ µ (1 – n) mg = n mg
a= – 10(0.26) = 7.5 – 2.6 m/s2 =4.9m/s2
100
 n 
µ=  
Sol 20: (B) Here in the problem, two cases arises; 1 – n

(i) when the body is at rest


4 . 8 2 | Forces and Laws of Motion

Circular Dynamics mv 2
Sol 26: (C) Centripetal force =
R
Sol 22: (D) Force acting on the particle at any instant is 5
v = 36 km/hr = 36  1000 m / s  = 36
 
mRω2 towards the center. m/s
→  3600  18
ds v = 10 m/s
2
F = (500)(10)
P 50
F F = 1000 N

i.e. F = mRw2 êr [ Radial dire ction]
Sol 27: (B) Use F = mrω2
And the displacement of the particle will be ‘ds’ along
tangential direction. Sol 28: (A) N
→ P
i.e. d s = ds êt
→ → θ
Now work = F . d s A
mg
W = mrw ds ( eˆ r .eˆ t )
2
θ

W = Zero (As eˆ r ,eˆ t are perpendicular to each other) O

Hence the work done by the Centripetal force is zero.

mv 2
Sol 23: (C) Centripetal force = mRw2 N = mg cos θ –
R
C
As one goes from A to P; θ decreases, so cos θ increase.
R ∴ N increases
D B

k
A Sol 29: (A) Centripetal force = –
r2
mv 2 k
Now at any point in the circle this value remains the ⇒ =–
same. Its only that the direction keeps changing. r r2
k
⇒ mv2 = –
Sol 24: (D) In uniform circular motion, ω is constant r
Now in the options, A, B, C the quantities are constant 1 k
⇒ mv2 = –
in magnitude but keep changing in direction. 2 2r
And since they are vector Quantities, we can’t say they k
⇒ kinetic energy K = –
are constant. For speed, its only magnitude that matters. 2r
Since it’s a Scalar Quantity. And since the motion is horizontal motion; let us assume
And Speed = Rω  Constant the potential energy same as that of ground i.e. zero
k
Hence option D. ∴ total energy = K + U = – +0
k 2r
E=–
Sol 25: (A) m1 = m2 = m ; v1 = v2 = v 2r

m1 v12 mv 2 Sol 30: (B)


Now F1 = = N cosθ
r1 r1 N
m2 v 22 mv 2
F2 = = mv 2
r2 r2 N Sinθ=
R
F1  r2 
=   θ
F2  r1  mg
P hysi cs | 4.83

mv 2 mvB2
N sin θ = TB = mg cos θ +
R R
N cos θ = mg mv 2A mvB2
Now TA – TB = –
v2 R R
tan θ =
Rg m 2
1 TA – TB = ( v – vB2 )  (i)
θ = tan–1= tan–1 =tan–1   R A
5 Now using conservation of energy theorem;
1
Sol 31: (D) At point A; EA = mv 2A + UA
2
v2
1
At point B; EB = mvB2 + UB
2
B EA = EB
T
1
v1 m( v 2A – vB2 )= UB – UA
A 2
mg But we can observe that both points A and B are at
same heights from the center.
In a vertical motion, the speed of ball doesn’t remain m 2 2
∴ UA = – UB  TA – TB = . (U – UA) = (UB – UA)
constant and as we discussed earlier, centripetal force R m B R
can’t be constant in direction itself, so its ruled out.
∴ is constant
And for tension, consider two points A, B
Sol 33: (D)
→  mv 2A  ˆ → mvB2
T A =  mg +  (– j ) and T B = ( î )
 R  R mv 2
 
R
Hence tension is also not constant. Now gravitational
force on the ball is (mg) at any point on the circle.
N
mg
Sol 32: (B) At point A

mv 2A
R
mv 2
mg cosθ mg + N =
R
TA
R
v= (mg + N)
A y m
θ θ Now for minimum case; let us say he just loses contact
mg
x i.e. N = 0

B ∴v= gR . This is the minimum speed.


θ
mg
mv 2A Previous Years’ Questions
TA + mg cos θ =
R
mv 2A Forces and Laws of Motion
TA = – mg cosθ
R
and for point B, F 5 × 10 4 5
TB Sol 1: (C) a = = 7 = × 10–3 m/s2
m 3 × 10 3
Mg cosθ
mvB2 5
v= 2as = 2 × × 10 –3 × 3 = 0. 01 m/s
R 3
4 . 8 4 | Forces and Laws of Motion

Sol 2: (A) Since, mg cos θ > mg sinθ Therefore,


∴ force of friction is f = mg sinθ Ft = ma = mαL = N
∴ Limiting value of friction
Sol 3: (C)
(fr)max = mN = mmαL  … (i)
Angular velocity at time t is ω = at
θ
T ∴ Centripetal force at time t will be

θ Fc = mLw2 = mLa2t2  … (ii)


Equating equation (i) and (ii), we get
µ
mg t=
α
mv 2
FBD of bob is T sinθ = µ
R For t > , F > (fr)max i.e. , the bead starts sliding.
and T cos θ = mg α c
v2 In the figure Ft is perpendicular to the paper inwards.
(10)2
∴ tan θ = Rg =
(10)(10)
Sol 6: (A) Since, the block rises to the same heights in
tan θ = 1 or θ = 45° all the four cases, from conservation of energy, speed of
the block at highest point will be same in all four cases.
Say it is v0.
Sol 4: (A) N = 5N
(f)max = mN= (0. 5)(5) = 2. 5 N V0

5N N + mg

N Equation of motion will be


mv 20 mv 20
N + mg = or N= – mg
w = 0.98 N R R
R (The radius of curvature) in first case is minimum.
For vertical equilibrium of the block Therefore, normal reaction N will be maximum in first
F = mg = 0.98 N < (f)max case.
Note in the question it should be mentioned that all the
Sol 5: (A) Tangential force (Ft) of the bead will be given four tracks are frictionless. Otherwise, v0 will be different
by the normal reaction (N), while centripetal force (Fc) in different tracks.
is provided by friction (fr). The bead starts sliding when
the centripetal force is just equal to the limiting friction. Sol 7: (A) Equilibrium of insect give
ω N = mg cosa
mN = mg sina
L Ft
A x α
Fc µN N

α
Ft is inwards mg sinα
mg cosα
mg

From Equation (i) and (ii). We get


cotα = 1/µ = 3
P hysi cs | 4.85

Sol 8: (D) Sol 10: (A) Free body diagram (FBD) of the block (shown
by a dot) is shown in figure.
T=Mg
N Vertical

T= Mg
f
F cos 60° Horizontal
mg

Free body diagram of pulley is shown in figure. Pulley is


mg+ F sin 60°
in equilibrium under four forces. Three forces as shown
in figure and the fourth, which is equal and opposite to
the resultant of these three forces, is the force applied For vertical equilibrium of the block
by the clamp on the pulley (say F). F
N = mg + F sin 60° = 3 g + 3  ... (i)
2
Resultant R of these three forces is For no motion, force of friction
R = ( (M + m)2 + M2 )g f ≥ F cos 60°
Therefore, the force F is equal and opposite to R as or mN ≥ F cos 60°
shown in figure.
1 3F F
or ( 3g + )≥
∴ F = ( (M + m)2 + M2 )g 2 3 2 2
F
F
or g ≥ or F ≤ 2g or 20 N
Mg 2
Therefore, maximum value of F is 20 N.
R
Sol 11: (D) This is the equilibrium of coplanar forces.
Mg+mg Hence,
ΣFx = 0
Sol 9: (C) Free body diagram of m is
T ∴ F=N
∴ ΣFy = 0, f = mg
 
Stc = 0 ∴ τN + τf = 0

∴ Since, τf ≠ 0

mg ∴ τN ≠ 0

T = mg  ... (i) Sol 12: (A) Initially under equilibrium of mass m


Free body diagram of mass 2 m is T = mg

T T Now, the string is cut. Therefore, T = mg force is


θ θ decreased on mass m upwards and downwards on mass
2m.
2m mg
∴ am = = g (downwards) and
m
mg g
a2m = = (upwards)
2m 2
2 mg
2T cos θ = 2 mg  … (ii)
Dividing Eq. (ii) by Eq. (i) we get
1
cos θ = or θ = 45°
2
4 . 8 6 | Forces and Laws of Motion

Sol 13: (B) y and T cos θ = mg


v2 (10)2
∴ tan θ = =
Rg (10)(10)
N
tan θ = 1 or θ = 45°
θ

x Sol 17: (D)


mg 90°– θ

θ
N sin θ = mg
N cos θ = ma
g T
tan θ =
a θ
a dy
cot θ = = tan(90° – θ) = = 2kx
g dx C
a
∴x= mg
2kg r

R =  sinθ
Sol 14: (A) When
T cosθ component will cancel mg.
P = mg (sin θ – µ cos θ)
T sinθ component will provide necessary centripetal
F = µ mg cos θ (upwards)
force to the ball towards centre C.
when P = mg sin θ
∴ T sinθ = mrw2 = m ( sinθ) w2
f=0
or T = mlw2
and when P = mg(sin θ + µ cos θ)
T
f = mmg cos θ (downwards) ∴ ω=
m
Hence friction is first positive, then zero and then
negative. Tmax 324
or wmax = = = 36rad/s
m 0.5 × 0.5
∴ Correct option is (A).

Sol 15: (B,D) A rotating/revolving frame is acceleration −bt


and hence non-inertial. Therefore, correct options are (B) Sol 18: (C) F = F0 e
and (D). F F0 −bt
⇒ a == e
m m
Circular Dynamics
dv F0 −bt
Sol 16: (C) ⇒ = e
dt m
t
F −bt
θ
⇒ ∫ dv = ∫ me dt
T 0

F  −1   −bt 1
θ ⇒ v=   e 0
m b 

F  −bt 
mg ⇒ v= e
mb  
mv 2
FBD of bob is T sinθ =
R
P hysi cs | 4.87

=v 0=at t 0 In this case,

F N
and v→ as t → ∞
mb u = 10 m/s
man
So, velocity increases continuously and attains a a=0
F
maximum value= of v as t → ∞ mg
mb
N – mg = 0 ⇒ N = mg = 70×10 = 700 Newton.
Sol 19: (C) mg sin θ = µmg cos θ
tanθ = µ ⇒ reading by the scale = 70 kg
Case (II)
dy 1
⇒ = tan θ = µ = In the frame of the lift;
dx 2
ma
x2 1 1 N
⇒ = , x = ±1 ⇒ y= m (pseudo
2 2 6 force) man a = 5 m/s2

Sol 20: (B) Normal force on block A due to B and mg


between B and wall will be F.
Friction on A due to B = 20 N ⇒ N + ma = mg

∴ Friction on B due to wall = 100 + 20 = 120 N ⇒ N=m(g−a)


⇒ N = 70 (10 – 5)
Sol 21: (B) Since work done by friction on parts PQ and
⇒ N = 70 × 5 N
QR are equal
N = 350 Newton
3 (QR=x)
−µmg × × 4 = −µmgx ⇒ Reading by the scale = 35 kg
2


= x 2 3 m ≈ 3.5m Case (III)
N
Applying work energy theorem from P to R
m a
0 3
mg sin 30 × 4 − µmg × 4 − µmgx = 0
2
1 mg ma
⇒=µ ≈ 0.29
2 3 N = mg + ma
⇒ N = m(a + g)
⇒ N = 70 (10 + 5)
JEE Advanced/Boards
⇒ N = 70 (15)
Exercise 1 N = 1050 Newton
Forces and Laws of Motion ⇒ reading by the scale = 105 kg
Now In this case a = g downward,
Sol 1: The reading shown by the weighting scale is the
normal reaction between the man and the weighing ∴ from case (b);
scale. ⇒ N = m (g – a)
Now, in Case (I) ⇒ N = m(g – g)
N=0
i. e the man is in free fall.
4 . 8 8 | Forces and Laws of Motion

Sol 2: T – mg = ma
T m2 T = m (g + a)
Here rope tries to pull
T T = 40 (10 + 6)
the man down.
T = 640 N
T
m But Tmax = 600 N, hence the string breaks.

Case b:
N
T T
m1 T = m1g T + m2g = N a =4 m/s2

m1g T m2g mg

mg – T = ma
⇒ N = (m1 + m2)g …(i)
T = m(g – a)  … (i)
In case II;
T = 40 (10 – 4)
Now rope pulls the man up
T = 40 × 6
N T=240N
T
T
T < Tmax
T m2 m1 m1 g = T
Case c:
m1 g m2 g u = 5 m/s uniformly i.e. a = 0
T
T = mg = 40 (10)
m1
T = 400 N
T + N = m 2g
T ≤ Tmax
⇒ N = m2g – T
⇒ N = m2g – m1g Case d:

⇒ N = (m2 – m1)g In this case;

Hence normal force is less in second case. Put a = g in case (b)


We get t = m (g – a)
Sol 3: In climbing the rope, monkey tries to pull down
T = m (g – g)
the rope, and the rope pulls the monkey upwards.
T=0

Sol 4: Now with respect to the truck; forces on the mass


m =40 kg ‘m’ are
μ = 0.15
∴ On monkey; T ma (pseudo force)
a
5m

mg

Now in Case (a) N


T
m ma
a =6 f

mg mg
P hysi cs | 4.89

ma – f = ma’ ∴ Force on the floor by the crew is 7500 N downwards.


N = mg
Sol 6:
And f = mN = mmg µ= 0.18
ma – mmg = ma’
a = 0.5 m/s 0<t ≤ 20
a’=a−µg
a’ = 2 – (0.15) (10) a=0 t > 20
A’ = 2 – 1.5
For an observer on ground, this is how he depicts the
A’ = 0.5 m/s2 FBD of mass,
Now this fairly a relative motion problem
m
Box has to cover a distance of 5 m to fall off from the f
truck;
1 1 f = ma
s = 0t + at2 ⇒ 5 = (0.5)t2
2 2
Now let us check for any sliding.
T= 20 s
f ≤ fs … (i) [Condition for no sliding]
Now in the meantime, distance traveled by the truck in
fs = mmg = (0. 18) (15 × 10) = 27 N.
1
s = (2)(20) =20m and f = ma ≡ 15(0. 5) = 7. 5 N.
2
hence no sliding.
∴Distance from the starting point where the box lands
is 15m. The observer will find the body to move with acceleration
of 0. 5 m/s.
Sol 5: Now since there is no sliding, there is no relative motion
F
w. r. t. the trolley.
Helicopter
Hence observer on trolley will find the mass to be at
M 15 m/s2
rest.
m Crew
k1 k2
Sol 7: m
(M + m)g
F is the force on helicopter due to the surrounding air
∴ F – (M + m)g = (M + m)a k1 k2
m
⇒ F = (M + m)g + (M + m)a
⇒ F = (M + m) (g + a) x
⇒ F = (1300) (25) N = 32500 N upwards k1x k2x
m
Now using newton’s third law, force by helicopter on
surrounding air is F downward, i.e. 32500 downwards. k1x + k2x = ma

Now if we consider the crew, (k1 + k 2 )x


a=
N m

mg ma

⇒ N = m (a + g) = 300 (25)
N = 7500 N upwards.
4 . 9 0 | Forces and Laws of Motion

Sol 8: T2

T2sinθ A T1

T1 mg
a C T1
T2 cos θ = mg  … (i)
B T2 sin θ = T1  … (ii)
a
T2 T2=mg sec θ
T2 T1
From (ii) & (i) ⇒ tan θ =
A mg
⇒T=mgtan θ
mg Now just after the string AB is burnt,
T1 T2 = mg cos θ

a C
θ
mg
T2
T1 – mg = ma  … (i)

T1 θ
mg cos θ
a B mg

T2
Sol 10:
mg
T d T

T2 + mg – T1 = ma  … (ii) h
T
θθ T
T2

a A
d
mg θh
mg – T2 = ma  … (iii)
now, (i) + (ii) 2T cos θ = mg
gives T2 = 2ma mg
=T sec θ
2
Now using this is equation (iii)
2
g 2g 2 d
a= and T2 = h + 
3 3 2
sec θ =
h
O
Sol 9: mg h2 + (d / 2)2
∴T= .
θ 2 h
T2 cos θ
∴ We can see that when h decreases, T increases.
T2
T2 sin θ T1
m A
B
P hysi cs | 4.91

Sol 11: FBD of A; N2 = mg cos a  … (iv)


N m2g sin α – N – f2 = m2a … (v)
ma
f2 = k2N2  … (vi)
A 2 m/s2
A Now (i)/(v)
B
mg N f N f
g sin α+ – 1 = g sin α – ,– 2
m1 m1 m2 m2
mg = N + ma
Solving; f1 = k1m1 g cos a
⇒ N = m (g – a)
f2 = k2 m2g cos α
⇒ N = 0.5 (10 – 2)
1
⇒ N = (8) = 4 newton. gcos α(k1 – k 2 )m1m2
2 ∴ N=
(m1 + m2 )
Sol 12: Initially, Adding (i) + (v)
FB + FA = mg  … (i) (m1 + m2) g sin α – (f1 + f2) = (m1 + m2) a
But for just sliding case, a = 0
FB
(m1 + m2) g sin α = f1 + f2
v f1 = k1N1; f2 = k2 N2
m
FAir resistance = kv ∴ (m1 + m2 g sin α = k1m1 g cos α + k2 m2 g cos α
mg
k1m1 + k 2m2
Let us say mass ‘m’ is removed to achieve case b; finally; tan α =
m1 + m2
Fair = kv FB
Sol 14: mg sin α – f = ma
v
n f = µ mg cos α [ µ N]

(M–m)g f

(M – m )g + FA = FB  … (ii) mg sin α
From equation (i) and (ii), eliminating FA;
2(Mg – B) α
We get m =
g

mg sin a – µ mg cos α = ma
Sol 13: a N1 N
2
f1 1 ∴ a = g sin α – mg cos α
1
mg1 sin α a = g(sin α – µ cos α)
m1g cos α
Now time taken by the block to reach point O;
m1g sin α + N – f1 = m1a  … (i) 1
s= ot + at2 ∴ s =  cos α
N1 = mg cos α  … (ii) 2
1
f1 = m1N1 = k1N1  … (iii)  cos α = g (sin α – µ cos α) t2
2
N2 2 cos α
t=
f1 g(sin α – µ cos α )
N 2
m2g sin α for minimum t;
dt
m2g sin θ = 0.

4 . 9 2 | Forces and Laws of Motion

1  –1  Sol 16: First let us calculate the limiting friction on


We get α = tan–1   blocks ‘A’ and ‘B’.
2  µ 
µ B A µ
Sol 15: f cos α = ma  … (i)
f sin α + N = mg  … (ii) fsA = µ mg

F sin α fsB = µ mg
3
F Now when a force of µ mg acts on the block A; it
4
α
F cos α doesn’t cause any motion in A.
3
Hence; F = fA = µ mg
4
mg
And string is left unaltered. Hence tension is zero. And
Now at the moment, contact is lost;
hence fB = T = zero
N=0
3
(b) Now when force of µ mg is applied,
F sin α = mg 2
Body A will tend to move forward. (F ≥ fs)
at0sin α = mg
Let us assume that the whole system moves with on
 mg  acceleration ‘a’.
t0 =  
 asin α  a
dv
now F cos α = ma ≡ m
dt
dv T T
∴ at cos α = m B A F
dt
fB fA
Integrating on both sides
t0 v
On body A;
∫ (acos α)t dt = m∫ dv
0 0 a

acos α t02 N
. = vm … (i)
2 2
T F
acos α m2g2 mg2 cos α fA
⇒v= . ⇒ v =
2m a2 sin2 α 2asin2 α mg

We see that in equation (i) F – T – fA = ma  … (i)


acos α 2 mg = N  … (ii)
⇒v= t
2m fA = µ mg  … (iii)
dx acos α 2 On body B;
v= = t
dt 2m N
acos α 2 T
dx = t dt
2m fB

Integrating on both sides; mg

x t0 t0
acos α 2 acos α t3 T – fB = ma  … (iv)
⇒ ∫ dx = ∫ 2m
t dt ⇒x=
2m 3
0 0 0 fB = µ mg  … (v)
3
 acos α   mg  Adding (i) and (iv);
x=    
 6m   asin α  F – (fA + fB) = 2ma
P hysi cs | 4.93

3 µ mg – (2 µ mg) = 2 ma 13
⇒ t2 =
2 2(2 – 3µ )g
a is negative
1 2
Distance travelled by B = at
It means that our assumption that both the bodies 2 B
move is false.
1 µg 13 13µ
= × . =
T F 2 4 2(2 – 3µ )g 16(2 – 3µ )
fA

F – T – fA = 0 Sol 18: mass m2 = hm1

3 Friction force on m1 f = km1 g cos a


T = F – fA= µ mg – µ mg
2 Gravitational force on m1, f1 = m1 g sin α
mg
T= µ m2g – f – f1
2 acceleration a =
m1 + m2
Now we can see that
ηm1g – km1gcos α – m1gsin α
=
T m1 + ηm1
fB
µ mg g(η – k cos α – sin α )
T = fB = ≤ fs ≡ µ mg a=
2 (n + 1)

∴ Block B cannot move. Since they both are connected


to each other, even A can’t move. Sol 19: By constrains of string,
Acceleration of A equals to B

Sol 17: Length of block A =
4 ⇒ aA = ac = a
⇒ Distance travelled by A relative to B ⇒ (mA + mC) a = (mA + mC) g sin θ – µ (mC) g cos θ
3 1
= +   ⇒ 2 ma = 2 mg sin θ – µ mg cos θ
4 44
1 3
13 a= (2g sin θ – tan θ g cos θ) = g sin θ
l0 = 2 4
16 3
∴ aA = aC = g sin θ
Let mass of A be mA = m 4
Now for B, tensions of string cancel each other and no
mc = m; mB = 4m friction exists.
friction force = μmAg Hence the only acceleration is due to gravity
f µmA g µg
Acceleration of B aB = = = ∴ aB = g sin θ
mb mB 4

mc g – µmA g Sol 20: m will have acceleration vertically downword.


Acceleration of A aA = Let call it am.
mc + mA
mg(1 – µ ) (1 – µ ) M will have acceleration along inclined plane lets call
= = g is aM
2m 2
Relative acceleration a = aA – aB  m, M have no relative acceleration vertically
downward,
g(1 – µ ) µg g
= – = (2 – 3µ ) aM sin α = am  … (i)
2 4 4
1 2 Let normal force on block be N,
at = l
2 mg – N = m am
1 g 13
∴ (2 – 3 µ) t2 = N = m (g – am)
2 4 16
4 . 9 4 | Forces and Laws of Motion

From free body diagram of wedge N d


⇒ (cos β + µ sin β) = 0

Mg sin α + N sin α = M am
M
∴ mg sin α + m(g – am) sin α = Mam ⇒ – sin β + µ cos β = 0
mg sin α + (m (g – aM sin α ) sin α=M am M g sin α ⇒ µ = tan β
1 µ
(m + M)gsin α ⇒ β = tan–1 µ ⇒ cos β = 2 and sin β =
⇒ am = µ +1 µ2 + 1
M + msin2 α
mg(sin α + µ cos α )
(m + M)gsin2 α
⇒ am = (am = aM sin α) 1 µ2 mg(sin α + µ cos α )
M + msin2 α T= + =
2
µ +1 2
µ +1 µ2 + 1

Sol 22: Let mass of A = m


Mass of B = hm Sol 24: frictional force f = km2g

Let normal reaction between surfaces be N F–f


a1 = (F > f)
m1
Nsin α Nsin α
aB = = at − km2g
mB ηm a1 =
m1
mg – Ncos α
aA = at – km2g
m a1 = (t > t0)
m1
aA = aB tan α km2g
a2 = = kg (t > t0)
Nsin α tan α m2
N
⇒g– cos α =
m ηm for t < to , f acts as internal force as there is no sliding
at
N sin α tan α  ∴ a1 = a2 = (t < t0)
⇒g=  cos α +  m1 + m2
m η 

N g a1 a2
⇒ =
ηm η cos α + sin α tan α
N g t
aB = sin α = t t0
ηm η cot α + tan α t0
g Till time to , the bodies move together.
aA = aB tan α = 2
η cot α + 1 at t = t0, f = k m2g
km2g = m2a2
m2
Sol 23: Let tension in string be T
kg = a2 = a1 m1
Net force perpendicular to plane
ato – km2g = m1a1
N = mg cos α – T sin B m2 f = k m2g
ato = km2g + m1a1
For minimum tension acceleration is zero k(m1 + m2 )g
to = f m1 F = at
∴ mg sin α = T cos β – µ N a
mg sin α = T cos β – µ mg cos α + mT sin β
Sol 25: Let mass of motor = m
mg(sin α + µ cos α )
T= mass of bar = 2m
cos β + µ sin β
2m w = T – 2mg k
dT mg(sin α + µ cos α ) d
= (cos β + µ sin β) ⇒ T = 2 m (w + kg)
dβ –(cos β + µ sin β)2 dβ
Let acceleration or motor be am
dT
For minimum T, =0 m am = T – mg K

P hysi cs | 4.95

1 300 × 2
⇒ am = [2mw + 2m kg – mkg] ⇒ aB = = 1.558 ms-2
m 385
⇒ am = 2w + kg 3
aA = a = 2. 338 ms2
2 B
Relative accelerator a = am + w = 3w + k g
T = 81. 8 N
1 2
at = l
2
2 2 Sol 28: F = 30t N
t= = 3w + kg
a ⇒ T = 10 t
wt. of A = 10 m1 = 10 N
Sol 26:
F (a) Block A loses contact when T = weight

10t = 10
 θ t=1s
T
Similarly 2T = 10m2 when B loses contact
m
20t = 10(4)
2T cos θ = F
t = 2s
F
⇒T= (b) Net force on A FA = 10 t – 10 (t > 1)
2cos θ 1
aA = (10t – 10)
T sin θ m1
Horizontal acceleration ax =
m
aA = (10t – 10)
F sin θ F tan θ F x
= = = . dv A
2cos θ m 2m 2m ( )2 – (x)2 = 10t – 10
dt
Fx 2 2
ax = 1 vA = ∫ (10t – 10).dt = 5t2 – 10t
1
2m(2 – x2 )2 1

fx v = 5 ms–1
Acceleration of approach = 2ax = 1 t t
2
m(2 – x2 ) 2 (c) vA = ∫ (10t – 10)dt = 5t – 10t
1
1

Sol 27: Let tension in thread = T vA = 5t2 – 10t + 5


F – 3T = 35 aB dh
= 5t2 – 10t + 5
2T = 70 aA dt
2 2
⇒ F = 35 (aB + 3aA) 2 5 3 5
H = ∫ dh = ∫ (5t – 10t + 5)dt = 3
t – 5t2 + 5t =
3
m.
Constrain equation xB 1 1

xA xB Circular Dynamics

Sol 29: Acceleration inside a rotor = Rω2



xc a = Rω2

Now for a max
2(xB – xA) + (xB – xC) = Constant
2
3 amax = R wmax
⇒ 3aB – 2aA = 0 ⇒ aA = aB
2
Given amax = 10g = 100 m/s2
 11 
⇒ F = 35   aB
 2 
4 . 9 6 | Forces and Laws of Motion

Sol 31:
100 10
ωmax = = rad/s = 5 rad/s θ
4 2

1
we know that 1 rad = rev

5
∴ ωm = Rev/s

T cos θ = mg
Sol 30: 2
T sinθ = mv
N R
θ
Now the component T cosθ has to balance the weight
θ mv 2 of the body
R
∴ Tmax cosθ = mg ⇒ 8 cosθ = 0. 4 × 10
θ 1
cosθ = ⇒ θ = 60°
2
mg ∴ Angle with the horizontal is (90°– θ)= 30°
mv 2
and T sinθ =
mv 2 R
N sin θ =  ... (i)
R
3 0.4 × v 2
N cos θ = mg  ... (ii) 8. =
2 4
Dividing (i) and (ii) v= 40( 3) m/s
v2 v = 8. 3 m/s
⇒ tanθ =
Rg
5 Sol 32: Speed of the particle just before the string
⇒ v = 108 km/h = 108 × m/s
18 breaks is v. Now after the string is broken; the path of
v = 30 m/s the stone will be;
R = 90 m v
30.30
∴ tanθ = =1
90.10
π 2
⇒ θ=
4
Squaring (i) and (ii) and adding them
2 10
 mv 2 
⇒ N (sin θ + cos θ) = 
2 2
 + (mg)2
2
Writing the equations of motion;
 R 
 
1 2
2
along y : 2 = gt  ... (i)
 mv 2  2
2
⇒N= (mg) +  
 R  along x : 10=vt  ... (ii)
 
Solving for v; we get
⇒ N = m (10)2 + (10)2 = 10 2 m Newton v = 15.8 m/s
v2
and centripetal acceleration =
⇒ N = 10 . 4
2 N. R
(15.8)2
a= = 168.3 m/s2
1.5
P hysi cs | 4.97

Sol 33: Sol 35: mv 2


cosθ
B R
v N
mv 2
R
θ
mgcosθ
mg sinθ
mg mv 2
u sinθ
A R

Writing down the equation of motions at point A and B; mv 2


N= sin θ + mg cos θ
R
At B:
mv 2 mv 2
TB = – mg f= cosθ – mg sin θ
R R
mu2 Contact force is N + f
At A:- TA = + mg
R
And the angle with which the force and the surface of
Now for completing the circle; the contact lie is
Tension at the highest point has to be non-zero; or else f
tan–1  
the particle will fall down. N
So for the minimum case, T ≈ 0 But here given that the force is normal to the surface
∴ TB = 0 ⇒ Friction force = 0
2
mv mv 2
⇒ = mg
R ∴ cosθ – mg sin θ = 0
R
Rg v2  v2 
v= ⇒ tanθ = ⇒ θ = tan–1  
Rg  Rg 
 
5 250
v = 100 km/h = 100. = m/s
18 9
Sol 34:
Find θ now !

Sol 36: FBD of M;

T mv 2
R MLω2
mv 2 f
T=
R
Mg
Now for vmax
f = MLω2; N = Mg
we have Tmax = 500 N
and for static conditions;
1v 2
⇒ 500 =
1 f = mN = µMg ⇒ µMg = MLω2

v= 500 = 10 5 m/s µg
ω=
L
v = 22.36 m/s
4 . 9 8 | Forces and Laws of Motion

Sol 37: Let u be the speed at the highest point of the Let us call the point where tension is equal to the weight
bridge of the particle as ‘P’.
A
u Now at point P,
P
R θ mv 2
T= + mg cos θ  ... (i)
mg R
O Given that T = mg

mv 2
mg = + mg cos θ
R
mu2 mv 2
+ N = mg mg(1 – cos θ) =  ... (ii)
R R
mu2 Now Total energy at point O
N = mg –
R 1
0= m ( gL )2 + 0
Now for maximum speed where contact is broken; 2
N=0 mgL
2 E0 =
mu 2
∴ mg =
R 1
u= Total energy at point P = m(v2) + mgL (1 – cos θ)
Rg 2
E0 = EP

Sol 38: mgL mv 2


∴ = + mgL (1 – cos θ)
2 2
θ mgL mv 2
T – + mg L cosθ =
2 2
mv 2 mv 2
R = 2 mg cosθ – mg  ... (iii)
2
mg mv 2
Now using this value of in eqn (ii)
mv 2 L
T sin θ =  ... (i) 2 mg cos θ – mg = mg(1 – cosθ)
R
T cos θ = mg  ... (ii) 3 mg cosθ = 2 mg
2 2
cosθ = ⇒ θ = cos–1  
(i) v2 3 3
= tanθ = Considering eqn – 3
(ii) Rg
5 mv 2 2
v = 36 km/h = 36 = 10 m/s = 2mg   – mg
18 L 3
10 × 10 mv 2 mg
tan θ = ; tan θ = 1 =
10 × 10 L 3
π g
⇒ θ= v=
4 3

Sol 39: Sol 40:


θ R
N

θ r
mrω2
T
v
P r=R sinθ
mg
θ mv 2

O gL FBD of body:
R
mg
P hysi cs | 4.99

(a) For minimum ω; Sol 42:


m1
Body tends to slip down w
F1
∴ friction acts upwards F2
m2
N
f
FBD of m1;
mrω2
a
T M1 F1 F1 – T = m1a  ... (i)
mg
FBD of m2 :
N = mg cosθ + mr ω sinθ 2
a
F = mg sinθ – mr ω2 cosθ M2 F2 T – F2 = m2a  ... (ii)
T
We know that f = mN
Adding equation (i) and (ii)
⇒ mgsinθ – mrω2cosθ=m[mg cosθ + mrω2sinθ]
F1 – F2 = (m1 + m2)a
Separating all ω2 terms to one side;
F1 = mRω2 F2 = 2 mRω2
(mr sinθ+r cosθ)ω2=g sin θ – mg cosθ
∴ – mRω2 = 3m a
g(sin θ – µ cos θ)
ω= Rω2
R sin θ(µ sin θ + cos θ) a=–
3
Now for maximum limit case; Using equation (i)
Solve exactly as above N  Rω2 
mRω2 – T = m  – 
 3 
mrω2  
mRω2
f T = mRω + 2

mg
3
4
T = mRω2
Sol 41: 3

θ Sol 43: Given Normal acceleration an = Kt2


mv 2
But we know that man =
2 R
∴ v = Kt2
R
Fcosθ
v= KR t  ... (i)
θ
F=mRω 2
dv
= KR
dt
dv
F sinθ
Tangential force = m. =( KR )m = m KR
→ dt
Total force = m| a |
Now mRω2 cosθ = ma → → →
a = an + a t
∴ a = Rω cosθ
2

1 2 →
Now s = ut + at |a|= an2 + a2t = (Kt 2 )2 + ( KR )2
2
1 →
L=0+ Rω2 cosθ t2 Total force = m. | a | = m K(R + Kt 4 )
2
Now we know that work done by normal force in a
2L circular motion is zero
t=
Rω2 cos θ
4 . 1 0 0 | Forces and Laws of Motion

∴ ωN = 0 Let us consider the part OAB;


Now only work is done by tangential force T cosθ/2
T
(m. KR )ds  ... (i)
T sinθ/2
We know that
dω dω ds θ/2
Power = ≡ . mR2ω2
dt ds dt θ/2

P = v. = KR × t × m KR
ds T sinθ/2
P = mKRt T
t T cosθ/2
∫ mKRt dt m is the mass of the part OAB.
0
Avg power = t
θ
⇒ 2T sin = mR2ω2
∫ dt 2
0
1 now for small values of θ; sinθ = θ;
Pavg = mKRt
2 θ
2T.   = mRω2
mv 2 2
Sol 44: A N R
Tθ = mRw2 ... (i)
Rsinθ

B ma Now m = (λ)(Length) = (λ). Rq
R cosθ θ
R
mg ∴ Tθ = (lRθ)Rω2
T = lR2ω2

a → → →
Sol 46: a net = a radial + a tangential
In this case, there will be a pseudo force acting on the →
v2 →
body. Now we use Work-Energy theorem, i.e. work done ar = . (– êr ); a t = a( êt )
R
by all the forces is equal to change in kinetic energy. We
know that, work done by normal force and centripetal → 2
 v2 
force is zero | a net |= 2
a +  m/s2
R 
 
Work done by pseudo force = ma. (R sin θ)
WPF = maR sinθ
Sol 47: B
Work done by gravitational force = mg(R – Rcos θ)
A
Wmg = mgR(1– cosθ) θ w
Net work done = maR sinθ + mgR (1–cos θ) O
1
≡ mv2 = Rm(a sinθ + g(1 – cosθ))
2
v= 2R(asin θ + g(1 – cos θ)) Consider the part OAB;
Let the mass of this strip be ‘dm’

Sol 45: T

θ θ/2 dmv 2
O A
θ/2 R

T
P hysi cs | 4.101

θ dm.v 2 The mass of the part AB of chain has to be supported by


2T sin   = the rest of the chain.
2 R
For very small values of θ; sin θ ≈ θ m
T = (L – x)g
L
θ dm.v 2 dm.v 2
∴ 2T  2  = ; T. θ =
  R R Sol 2: (A) mg sin θ = f and N = mg cos θ
m  mθ  mθ v 2 N
Now dm = .R.θ =   ⇒ Tθ = .
2πR  2π  2π R
f
2
mv
T=
2πR
at mg sin θ mg cos θ
Sol 49: → θ
a

mg
ar For the condition of just sliding;
f = mN

→ → →
⇒ mg sin θ = µ mg cos θ
a net = ar + at ⇒ tan θ = µ ⇒ θ = tan–1(µ).
2
→ v → dv
ar = ; at = =a Hence the angle of inclination has nothing to do with
R dt the mass of the body.
2
→ v2 →  v2 
a net = (– êr ) + a ( êt ); | a net | = a2 +   Here the angles are different because of the change in
R R 
  ‘µ’ from one block to another.

f = m| a net |
Sol 3: (A) a
∴ Under static conditions

2
F m2
2
 v2  m1 m3
mmg = m a + 
R 
 

v = [(m2g2 – a2)R2]1/4
Now let us say the whole system moves with an
acceleration ‘a’.

Exercise 2 ∴ F = (m1 + m2 + m3) a … (i)


Let as consider Individual masses;
Forces and Laws of Motion
For m1 ;
Single Correct Choice Type Ng
1

m1 F – N1 = m1a; …(ii)
F N1
Ng – m1g= 0; …(iii)
1
Sol 1: (C) A x m1g
(L–x) a
B
m For m2:
λ (linear density) of chain =  
L
Now at point A; f
N1 N2 N1 – N2 = m2a; …(iv)
T m2g – f = 0; …(v)
m2 g
m
  (L – x)g
L a
4 . 1 0 2 | Forces and Laws of Motion

For m3; m1g sin θ + N – fs = m1a


1
Ng
3 N = m1a – m1g sin θ + fs
1
N2 N2 N2 = m3a; …(vi)
 35 
N = 170   – 800 + 300
Ng = f + m3g; …(vii)
3  17 
f m3 g N = – 150 N
a
i.e. force in the bar is 150 N.
We know that fmax = mN2 = mm3a
Sol 5: (A) Lift moving uniformly means lift is moving
f = m2g ≤ fmax ≡ mm3a without any acceleration.
m g Hence in both the cases; acceleration of the coin is ‘g’.
⇒ a ≥  2  … (viii)
 µm  ∴ t1 = t2.
 3 

m g
⇒ F ≥ (m1 + m2 + m3)  2  (from (viii) & (i)) T
 µm3  Sol 6: (B) m2

T 700 gm F
Sol 4: (A) µ 2 = 0.4 m1

Now if m1 moves with an acceleration ‘a’ towards right;


m2 will have an acceleration of ‘a’ towards left.

µ 1 = 0.2 [ string constraint]


FBD of m1;
θ N2 N1
f2
Here both the particles are constrained to move m1
T F
together. Hence aA = aB
f1
Now let us first find the net force down the incline;
i. e (m1 + m2)g sin θ m1g
8
Fnet = 340 × 10 ×
17 F – f1 – f2 – T = m1a  … (i)
Fnet = 1600 N.
m1g + N2 = N1  … (ii)
Now let us calculate the fs + fs
1 2
FDB of m2;
15
fs = µ1 . (m1g cos θ) = (0. 2) (170 × 10 × ) = 300 N N2
1 17

fs = µ . (m g cos θ) = 0. 4 (170 × 10 × 15 ) = 600 N T m2


2 2 2
17
f2
∴ fs1 + fs2 = 900 N.
∴ Net Acceleration of the system m2 g
 1600 – 900  700 35 N2 – m2g = 0  … (iii)
=   = m/s2 ∴a= m/s2
 170 + 170  340 17
T – f2 = m2 a  … (iv)
Now on A; N f2 = mN2 = mm2g  … (v)
a
fs
1
∴ T = m2a + mm2g
m1g sin θ
T = (a + μg) m2  … (vi)
f1 = mN1 = µ(m1g + m2g) = mg (m1 + m2)
P hysi cs | 4.103

∴In equation (i) Right.


F – mg (m1 + m2) – µ m2g – (a + mg)m2 = m1a ∴ FBD of m;
∴ F = (m1 + m2) a + 3mm2g + mm1g N ma

⇒ F = (m1 + m2)a + mg (m1 + 3m2) F m T


f
Put a = 0.3 m/s2 and m1 = 0.7 kg, m2 = 0.2 kg to get the
value of force. mg
Hence, we get F = 2.18 N
F–T–f=0
Sol 7: (A) N
N + ma = mg
m ⇒ F = f + T  … (i)
F1
N = mg – ma  … (ii)
mg cos θ
θ ∴ FBD of M;
N1 Ma
mg
f M T
As force F tends to push the mass upwards, friction will
tend to oppose it. So, it will act downwards.
∴ F = f + mg sin α Mg

f = m N = µ mg cos α
T – f = 0  … (iii)
⇒ F1 = µ mg cos α + mg sin α  … (i)
N1 + Ma = Mg  … (iv)
Now when pushing downwards, friction will be acting
From (i) and (iii);
upwards,
⇒F=f+f
N F2
⇒ F = 2f ; f = µN = µ(mg – ma)
m f
F = µm (g – a)
mg sin θ mg cos θ ⇒F = 2 mm (g – a).
θ
Sol 9: (B) a1
mg

∴ F2 + f = mg sin θ
F2 = mg sin θ – f m1 µ
f = µ mg cos θ µ= 0 m2 m F
⇒ F2 = mg sin θ – µ mg cos θ  … (ii)
Given that F1 = nF2
Let us say the whole system moves with an acceleration
∴ mmg cos θ + mg sin θ =n(mg sin θ – mmg cos θ)
a1.
n–1  F 
⇒ µ= tan θ ∴ a1 =    … (i)
n+1  m1 + m2 

Sol 8: (B) Now FBD of m1;


N
F T
m m1a1
f m1
M m f

m1 g
For maximum force, F; the friction on ‘M’ will be towards
4 . 1 0 4 | Forces and Laws of Motion

m1a1 – f= 0 And P – f = m1a ⇒ P – µ mg = m1a


⇒ m1a1 = f  … (ii) kt – µm1g
a1 =
FBD of m2 N m1
f
m2 F
m2a Sol 11: (B) Now let us check the limiting frictions
between the three surfaces
F – f – m 2a = 0
⇒ F = m2a + f  … (iii) (1) A P
µ1 = 0.3
Now when the mass m1 just tends to slide;
(2) B µ2 = 0.2
f = µ N = µm1g (3) C µ3 = 0.1
∴ m1a1 = µm1g (from (ii))
fs = µ1(mA g) = 90 N
∴ a1 = µg. 1
fs = µ2(mA + mB)g = 80 N
Now from (i) 2
fs = µ3(mA + mB + mC) g = 60 N.
F = (m1 + m2) µg 3

at = (m1 + m2) µg ∴ Now let us assume P would be greater than 60 N and


less than 80 N.
(m1 + m2 )µg
t= For this P;
a
f1 = P [ f1 < fs ≡ 90]
1
Sol 10: (C)
A p
N f1
m1 f1 = f2 = P [ f2 < fs ≡ 80]
m1 p P=kt
f A 2
f1
m2 B B
m1 g f2

p – f = m1a  … (i) Now f2 – f3 = m3a

N = m1g  … (ii) f2 [ f2 = P > 60 ≡ fs ]


C 3
f3
f
m2
∴Here f2 is greater than the maximum static friction
f = m2a  … (iii) between C and ground. Hence the block C will slide on
the ground. There by all the three blocks will slide for a
Now for f ≤ mm1g; minimum force of 60 N.
Both the block will move together;
Sol 12: (A) m2g – T = m2a1 … (i)
∴Adding (i) and (iii);
P = (m1 + m2)a.
 P   k 
a =   =   t
m
 1 + m2 
 m + m
 1 2 

Now for f = mm1g; this is the maximum frictional force; a


T
T
∴ f = m2a2 π/4
m2
µm1g = m2a2 m1

µm1g
⇒ a2 = which is constant
m2
P hysi cs | 4.105

On m1; mAg sin θ – f – T = mA aA … (iii)


Tsin θ
N2 = mAg cos θ  … (iv)
N
T cos θ m1 aA = aB (constraint equation)
f
By adding (i) and (iii)
m1g
(mA – mB) g sin θ – 2f = (mA + mB) a  … (v)
N + T sin θ = m1g
For limiting condition, a= 0.
T cos θ – f = m2a2
⇒ (mA – mB) g sin θ = 2f
Now for just initiating the motion;
Here f ≤ fs = mN1 = µ mB g cos q
a1= a2 =0
∴ m2g – T = 0  … (i) (mA – mB) g sin θ ≤ 2 µ mB g cos q

T cos θ – f = 0  … (ii) mA – mB
µ ≥ tan q
m2g cos θ = f  … (iii) 2mB

f = µN = µ(m1g – T sin θ) = µ(m1g – m2g sin θ) Now in equation (v)

⇒ m2gcos θ = µ(m1g – m2g sin θ) If m1 = m2;


Tension itself balances both the masses.
 m2 cos θ 
m =  
 m1 – m2 sin θ  So, no necessity for any friction.
C: we cannot explicitly say that. We
put θ = π/4.
need more information on µ.
D: when mA = mB;
Multiple Correct Choice Type
Put friction f = 0 in (i) and (iii)
Sol 14: (A, B)
And subtract them to get Tension ‘T’.

B Sol 15: (A, D) T sin θ


A

N1 A T cos θ
θ

FBD of B;
N1 mg f1
T

B
Acceleration of block will be zero. Since its constrained.
∴ mg + f1 = T sin θ … (i)
f mBg cos θ
mBg sin θ N1 = T cos θ  … (ii)
f1 f2
T – f – mBg sin θ = mB aB  … (i)
N1 = mBg cos θ  … (ii) N1 B N2
FBD of A;
N2
f T Mg
A
Mg – f1 – f2 = Ma  … (iii)
mAg cos θ N1 = N2 = T cos θ
mAg sin θ
f2 = µ(N2) = µ T cos θ
4 . 1 0 6 | Forces and Laws of Motion

f2 = µ N1 = µ T cos θ And adding equation (i) and (iii);


∴ From equation (i) (m1 + m2 )gsin θ
a=
mg + µ Tcos θ = T sin θ (m1 + m2 )

mg = T (sin θ – µ cos θ) a = g sin θ  … (v)

 mg  now using this we can find, N3;


100
T =  sin θ – µ cos θ  =
   3 N3 = zero, for all m1 and m2.
0.5 – 0.2  
 2 
 
Sol 17: (B, D)
T = 306 N.
Now using equation (iii) A
T
Mg – f1 – f2 = Ma B

Mg – 2 µ T cos θ = Ma
T
2µT cos θ
a=g– FBD of B;
M m2 a

a = 4. 7 m/s2 mg

N1 mg – T = ma  … (i)
Sol 16: (A, C) N3
A a
FBD of A; A
T
m1g cos θ
m1g sin θ mg sin θ

m1gsin θ + N3 = m1a1  … (i)


mg sin θ + T = ma  … (ii)
N1 = m1g cos θ  … (ii)
By adding (i) and (ii)
B mg + mg sin θ = 2 ma
A
mg
mg + = 2ma
θ fixed 2
3g
N2 ⇒ a=
N3 4
B Now using equation (i)
mg – T = ma.
m2g sinθ m2g cosθ T = mg – ma = mg –
3mg
4
m2g sin θ – N3 = m2a2  … (iii) mg
T=
4
N2 = m2g cos θ  … (iv)
now let us assume; a1 ≠ a2, then;
Assertion Reasoning Type
Both of them will lose contact
∴ N3 = 0. Sol 18: (A) Conceptual. Conservation of linear
momentum for a single particle do mean that the state
But we then find a1 = a2 = g sin θ.
of the body is conserved or constant unless an external
Hence both of them will have same acceleration. force acts on the body.
Now putting a1 = a2 = a
Sol 19: (D) Assertion: If the force is non-constant and
reverses itself over time, it can give a zero impulse.
P hysi cs | 4.107

For example: spring force would give a zero impulse


over one period of oscillation. C

α
Sol 20: (D) Here; weight of the book is because of the
Gravitational Attraction Between earth and book. There B T
will also be a gravitational force between book and ma
table, which is very small, hence always neglected.
That Gravitational force between table and book form Fnet
mg
an Action-Reaction pair.
ma
∴ = tan α ⇒ a = g tan a
Sol 21: (A) Both assertion and reason are statements of mg
Newton’s laws. Since its acting leftwards, the vehicle should move
→ rightwards.
Sol 22: (A) Momentum = m u .
We have to specify reference frame, because velocities Paragraph 2:
will vary in different frames. So, momentum which
In s’ frame;
implicitly depends on velocity might also very.
FBD of M. y
a
x
Comprehension Type
m
Paragraph 1:
M
θ
B
α
T2 cos α
a
α T2
A Nθ
T2 sin α
Ti
θ
mg
N1 Mg
T2 cos α = mg ; T2 sin α = T1
N sin θ = Ma  … (i)
⇒ T2 = mg seca; T1 = mg tana
N cos θ + Mg = N1  … (ii)
Now just after the string AB is cut;
FBD of m;
T2 = mg cos θ
N + ma sin θ
α
T2 B ma

mg sin θ + ma cos θ
mg cos θ
θ mg
mg sin θ
mg cos θ
mg N + ma sin θ = mg cos θ  … (iii)
Now when string BC is cut; mg sin θ + ma cos θ = ma’  … (iv)
Mass ‘m’ will just have force fall. Hence tension in string from (i) and (iii);
AB is zero.
N = mg cos θ – mg sin θ
Now suppose it is keep in a moving automobile;
Now in equation (i)
In automobile’s frame of reference, there is a pseudo
N sin θ = Ma
force acting on the mass.
(mg cos θ – ma sin θ) sin θ = Ma
The resultant force should be along BC.
4 . 1 0 8 | Forces and Laws of Motion

 mgsin θ cos θ  max cos θ


a=  2 
 M + msin θ 
max
Since its in –ve x direction; we add a ‘–’ sign.
 mgsin θ cos θ 
∴ a = – 
2
 M + msin θ  mg sin θ

Now using this value of a, solving equation (iv); ∴ max cos θ = mg sin θ
We get ax = g tan θ in positive x direction,
 mgsin θ cos θ 
mg sin θ + m   cos θ = ma’
2
 M + msin θ  Paragraph 3:
2
mgsin θ cos θ Given that the plank has very rough surface.
⇒ g sin θ + = a’
M + msin2 θ N
µ >> 0

Mgsin θ + mg sin3 θ + mgsin θ cos2 θ ∴FBD of A;


⇒ = a’ m T
M + msin2 θ f

Mgsin θ + mgsin θ(sin2 θ + cos2 θ)


⇒ a’ = mg
M + msin2 θ
T – f = 0 … (i) ⇔ T = f
Mgsin θ + mgsin θ (M + m)g sin θ
a’ = ∴ a’ = mg – N = 0 … (ii) ⇔ N = mg
M + msin2 θ M + m sin2 θ
FBD of B;
This is the acceleration of the block ‘m’ with respect to
T
the incline.
Force exerted by the mass ‘m’ on wedge is ‘N’. 2m
We can find this by; equation (iii)
∴ N + ma sin θ = mg cos θ 2mg

⇒ N = mg cos θ – ma sin θ T – 2mg = 0 … (iii) ⇔ T=2mg


 mg sin θ cos θ 
2
∴ f = 2mg
= mg cos θ – m  2


 M + msin θ  Net Contact force acting between block A and plank;

 Mmg  is N2 + f 2 = (mg)2 + (2mg)2


N= 
2 
 M + msin θ  F = mg 5
ax On the pulley;
N= T 2 = 2 2 mg.
m
mg sin θ T N
m
θ
T 2
T
Now in this question; the downward component of mg
sin θ has to be balanced.
Now just after this instant;
Normal reaction becomes zero.
P hysi cs | 4.109

On body A; Now for this the acceleration of the block should be


zero.
T
a1
mg sin θ = f + T
a2 m T f
120 – 90 = f
f = 30 N
mg mg sin θ
µ (mg cos θ) = 30
T = ma1 … (i) 1
µ ( 8 3 .10. ) = 30
2
mg = ma2.
3
µ= .
⇒ a2 = g 4
on body B
T Sol 38: (B)
2m a1
b µ = 0.8
2mg p
2mg – T = 2ma1  … (ii) At an angle θ;
From (i) and (ii) mg sin θ – f = 0
2g N – mg cos θ = 0 N
a1 = = 6. 66 m/s2 f
3 Now fmax = fs = mN.  m
Now aA = a1( î ) + a2 ( – ˆj )
mg sin θ mg cos θ
–2g ˆ ˆ θ
= i – gj
3 At this point the block starts sliding.

4g2 ∴ fmax = µ mg cos θ


|aA| = + g2
9 ∴ mg sin θ – µ mg cos θ = 0

13 ⇒ tan θ = µ
|aA| = g
3 ⇒ θ = tan–1(0. 8)

|aA| = 12 m/s2 θ = 40º


Now till this angle; f = mg sin θ
Paragraph 4: ∴ for θ = 30º,

Buoyant force = rgVimm = rgv f = mg/2

F = (0.9) (103).(10) (0. 2 × 1 × 1 × × 10–2) Now for θ = 45º, let us say body is not sliding mg sin
θ–f=0
FB = 18 N.
N = mg cos θ
Now T + FB = W +Fv. T FB
fs = mmg cos θ=mmg/ 2 = 0.8 (mg/ 2 )
T = W + Fv – FB A
 mg 
Fv = 60 N. f = mg sin θ =  
 2
T = 48 + 60 – 18 w Fv
But for our assumption;
T = 90 N.
f ≤ fs
 mg   mg 
⇒  ≤ (0. 8)  
 2  2
4 . 1 1 0 | Forces and Laws of Motion

which is not true. 1


∴ mv2 = as2
Hence the body would have started sliding 2

f = fs = µ N = µmg cos θ = µmg/ 2 2a


v= s
m
→ → →
Now a = a r + a t
Circular Dynamics
→ v2 2as2
 v2  ar = =
Sol 39: (A) Centripetal acceleration = rω2  or  R mR
 r 

Given that both have same periods. → dv dv ds
at = =
So ω1 = ω2 dt ds dt
a1 = Rω2 a2 = rω2 → dv
a t = v.
a1 R ds
=
a2 r
dv 2a
=
ds m
Sol 40: (A) Max Tension the string can sustain → 2as
⇒ at =
Tmax = 10 N. m
Mass of the sto → → →
1 a = ar + a t
ne = 250 gm = kg
4 2
 2as2   2as 2
Length of string = 10 cm = 0.1 m |a| = ar2 + a2t =   +
 mR   m 
T = mrω2  
T
0.1m
Tmax = 2
mr ωmax 2as s2
|a| = 1+
m R2
Tmax
ωmax = → s2
mr | F | = m |a| = 2as 1 +
R2
10
ωmax = 1 = 400 rad/s
× 0.1 Multiple Correct Choice Type
4
Sol 44: (B, C) 
wmax = 20 rad/s.
dv →
at
Given speed = v; and =a anet
dt
Sol 41: (D) Already discussed in Q. 40 So try this yourself → v2
ar = ê
r r O ar
Sol 42: (D) Let the angular speed of the thread is w. →
a t = a êt
For particle C T3 = mw 3l2 → → →
a net = ar + a t
For particle B T2 → T3 = mw22l → T2 = mw25l →
a net v2
= êr + a êt
For particle A T1 → T2 = mw2l → T1 = mw26l r
2
→  v2 
1 | a net |=   + a2
Sol 43: (B) Kinetic energy k = mv2  r 
2  

But given that k = as2 Now friction force f = m a net
P hysi cs | 4.111

2 T
 v2  θ
f=m   + a2 mrw2
 r 
 

2
 mv 2 
f=   + (ma)2 and f = mmg mg
 r 
 
Resolving into components
T cosθ = mg
Sol 45: (B, D)
T sinθ = mrω2 ; r = L + L sinθ
N cosθ ⇒ T sin θ = mω2 L (1 + sinθ)
N
T sin θ ω2L(1 + sin θ)
⇒ =
N sinθ T cos θ g
mg
θ
C
Sol 47: (B, D) F
N

Since µ ≈ 0, there would be no frictional force. D F F


B
N cosθ = mg
F
2
A
mv
N sinθ = Consider the figure, with force F on the particle at
R
different instants of time.
2
 mv 2  So it is evident that there should be some other forces
⇒N =   + (mg)2
 R  such that particle will have uniform circular motion
  → → →
2 ∴ F + F 2 =m a
mv
∴ N > mg as well as N >
R Since it’s a uniform circular motion

at = 0
Now when speed of the car is less than vc = 40 km/hr
and if we consider the frame of car; → → v2
∴ a = ar =
R
Both these forces are made equal through proper
banking. → →  mv 2 
∴ F + F 2 =  

mv 2c
cos θ  R 
mgsinθ → →
mv 2
R Now resultant of both the forces F and F 2 is which
r
Now if v < vcritical ,
mv 2 in turn keeps changing both in direction as well as
Then mg sinθ > cosθ magnitude.
R
⇒ It slips downwards. →
mv 2 ê →
∴ F2 = r–F
R

w Angle between êr and F keeps varying.
Sol 46: (A, B, C) L
θ L
Assertion Reasoning Type

So 48: (D) Concept of centrifugal force comes into


L sinθ picture only in a non-inertial frame. So, both of them
cannot co-exist in a same frame.
Free body diagram of mass m;
4 . 1 1 2 | Forces and Laws of Motion

Although it is true that they are equal and opposite they Sol 52: (B)
can’t cancel each other because of this. N

N cosθ
Sol 49: (C)
N θ

N sinθ
N cosθ = mg
θ mv 2
N sinθ =
R
So bending inwards is always essential. He does it so
2
as to get horizontal component of normal force as
N sin θ = mv
R centripetal force. Although bending lowers his center of
gravity, it’s not the reason.
N cos θ = mg
It is not the friction between the tyres that provide him Sol 53: (A)
centripetal force, but it is component of Normal force. N

Sol 50: (B) From the above solution;


mv 2
We can write N sinθ = θ
R
N cos θ = mg
m(2v)2 mv 2
Now when v is doubled, = 4. N cosθ = mg
R R
∴ Tendency is quadrupled mv 2
N sinθ =
R
 v2 
And also tan θ =   as v ↑ θ ↑ v2
 rg  tan θ =
  Rg

Sol 51: (E) when velocity is doubled


N cosθ (2v)2 V2
N tan θf = = 4.
Rg Rg
N sinθ Hence skidding tendency is quadrupled.

θ Sol 54: (D) Assertion is explained in Q. 46 and Reason is


true (It is conceptual)
mg
mv 2
N sin θ = Comprehension Type
R
Horizontal component of normal force provides the Paragraph 1:
centripetal force. Hence false.
mv 2
Reason:- R
A curved path need not always be circular path. In case
of elliptical paths, the force is not necessarily centripetal. N
P hysi cs | 4.113

At any instant, say speed is v. Normal force against wall, Paragraph 2:


2
mv
N=
R
Now frictional force, f = mN mv 2
N
R
µmv 2
F= (– êt ) [tangential]
R

And tangential acceleration say a t
Top view of the rotor
→ µmv 2
Now m a t = (– êt ) mv 2 µ.mv 2
R N= ; f = mN =
2 R R
a t = µv (– êt )

R For equilibrium;
fs
→ dv fs = mg
and also a t =
dt mN = mg
dv µv 2
=– µ.mv 2
dt R And this is = mg
R mg
dv µ
= – . dt Rg
v2 R v=
µ
Integrating both sides
v t v
dv µ 1  µt Match the Columns
∫ v 2
=–
R ∫ dt ⇒–   =–
R
v0 0  v  v0
Sol 61: mv 2c
1 1 µt R
= +
v v0 R C
mg
µv 2 T
Sol 57: (B) at = – T mvB2
R D
B R
dv dv dx dv
at = = . =v
dt dx dt dx
A u = 3 Rg
dv µv 2
⇒v =–
dx R At point B;
dv µ mvB2
=– dx TB =
v R R
Integrating both sides; And also total energy at point A;
v x
dv µ 1
E= m(u)2 + UA
∫ v
=–
R ∫ dx 2
v0 0
Now assume ground at the point A itself
 v  µx
ln   = – ∴ UA = 0
 v0  R
µx
– 1 9mRg
v = v0 e R EA = m (9Rg) =
2 2

And total energy at point B;


1
EB = m( vB2 ) + mg(R)
2
4 . 1 1 4 | Forces and Laws of Motion

According to conversation of energy 1


EB = mvB2 + mgR
EA = EB 2
1
1 9mgR EC = mv 2C + 2mgR
∴ mvB2 + mgR = 2
2 2
Now given that vA = 10
1 7
mvB2 = mgR
2 2 So; EA = EB (using conservation of energy)

vB = 7gR 1 1 2
m(10)2 = m( vB ) + mg(1). [R = 1, vA= 10, m = 1]
2 2
mvB2
and TB = = 7mg vB = 80 m/s
R
for point C; and similarly
mv 2c 1 1
Tc + mg = EA = EC ⇒ (10)2 = v 2C + g(2)
R 2 2
mv 2c vc = 60 m/s
Tc = – mg
R
∴ from (i), (ii), (iii)
Total energy at point C is TA = 10 + 100 = 110 N
1 TB = 80 N
Ec = mv 2c + mg(2R)
2
TC = 50 N
1
Ec = mv 2c + 2mgR ∴ minimum tension is 50 N
2
When string is horizontal i.e. at point B;
Ec = EA

1 9mgR
⇒ mv 2c + 2mgR = T mv 2
2 2
R
mv 2c 5mgR
= ⇒ vc = 5gR mg
2 2

mv 2c

vB2
∴ Tc = – mg = 5mg – mg = 4 mg
ar = = 80 m/s2 (– î )
R R

a t = g = 10 m/s2 (– ˆj )
Sol 62: VC C → → →
VB a net = ar + a t

D | a net | = (80)2 + (10)2
B

| a | = 10 65 m/s2

A VA At point C; tangential acceleration is zero

mv 2A vc
At point A; TA = mg +  ... (i)
R mg
mvB2
At point B; TB =  ... (ii) T
R
mv 2C
At point C; TC = – mg  ... (iii)
R
1
→ → v 2c
Energy at point A = mv 2A (point A is assumed to be ∴ a net = a r = = 60 m/s2
2 R
ground)
P hysi cs | 4.115

Sol 63: mv 2 Maximum friction between B and C = µ (mA + mB)g


N cosθ
R or f2 = 0.25 (3 + 4)(10) = 17 5 N
mv 2
R Maximum friction between C and ground
mg sinθ
f3 = µ (mA + mB + mC)g
θ mg
= 0.25(3 + 4 + 8)(10) = 37.5 N
mv 2
Block C and hence block B are moving in opposite
sinθ + mg cosθ
R directions with constant velocities and block A is at rest.
mv 2 Hence, net force no all three blocks should be zero. Free
N= sinθ + mg cosθ  ... (i) body diagrams have been shown below (Only horizontal
R
forces are shown)
Now depending on condition, friction can be upwards
f1 f2
or downwards.
B C T
For maximum speed, friction is downwards. T F
f2 f3
2
mv
∴f= cosθ – mg sinθ  ... (ii)
R For equilibrium of B
And also f = mN T = f1 + f2 = 25 N
 mv 2  mv 2 For equilibrium of C
⇒ µ sin θ + mgcos θ  = cos θ − mgsin θ
 R  R F = T + f2 + f3 = 80 N
 
 µ msin θ mcos θ  F
v2  –  = – (mg sinθ + mmg cos θ) Sol 2: Acceleration of rope a =
 R R  M
L
mg(sin θ + µ cos θ) F
v= M
M
m
(– µ sin θ + cos θ) A a B
R
a a
Rg(sin θ + µ cos θ)
vmax. = L–  
(cos θ – µ sin θ) T T
F
And for the minimum speed; A C C B
friction will be acting upwards Now to find tension at point C, a distance  from point
mv 2 B, we can write equation of motion of any one part (AC
∴ f = mg sinθ – cosθ  ... (iii)
R or CB), both moving with acceleration a.
And following the same argument
Equation of motion of part AC is
for f = 0
M F
 v2  T = (mass of AC) × (acceleration) = (L – )  
tanθ =   L M
 Rg   
  = F 1 – 
 L
v= Rgtanθ  ... (iv)
1
So 3: mg sinθ = (2)(10)   = 10 N = F1 (say)
2
Previous Years’ Questions
3
µ=
Forces and Laws of Motion 2kg 2

Sol 1: Maximum friction between A and B= mmAg


30°
or f1 = 0.25(3)(10) = 7.5 N
4 . 1 1 6 | Forces and Laws of Motion

 3  3 Equation of motion of M1
mmg cosθ =   (2) (10)   = 21.21 N = F2 (say)
 2  2  T + F1 – f1 = M1a  ... (i)
   
or T = 4a
(a) Force required to move the block down the plane
with constant velocity. Equation of motion M2

F2 F2 – T – f2 = M2a
F
v or 7.84 – T = 2a  ... (ii)
Solving eqs. (i) and (ii), we get

F1 a = 1.3 m/s2 and T = 5.2 N


30°
Sol 5: Constant velocity means net acceleration of the
F1 will be acting downwards, while F2 upwards. system is zero. Or net pulling force on the system is
zero. While calculating the pulling force, tension forces
Since F2 > F1, force required
are not taken into consideration. Therefore,
F = F2 – F1 = 11.21 N
(a) M1g = M2g sin37° + µ M2g cos37° + µ M3g
(b) Force required to move the block up the plane with
or M1 = M2 sin37° + µM2 cos37° + µ M3
constant velocity.
Substituting the values
F
v 3 4
M1 = (4)   + (0.25)(4)   +(0.25)(4) = 4.2 kg
5 5

F1+F2 (b) Since, M3 is moving with uniform velocity

30° T = µ M3g = (0.25)(4)(9.8) = 9.8 N

M3
F1 and F2 both will be acing downwards.
F = F1 + F2 = 31.21 N T
µM3g
Sol 4: Maximum force of friction between M1 and
inclined plane Sol 6: F
f1 = m1M1g cos θ = (0. 75)(4)(9. 8)(0.8) = 23.52 N θ

M1g sin θ = (4)(9.8)(0.6) = 23.52 N = F1 (say) m


µ
Maximum force of friction between M2 and inclined
plane Let F be applied at angle θ as shown in figure. Normal
f2 = m2M2g cos θ = (0. 25)(2)(9.8)(0.8) = 3.92 N reaction in this case will be
M2g sin θ = (2)(9.8)(0.6) = 11.76 N = F2 (say) N = mg – F sin θ
The limiting friction is therefore
f1
a fL = mN = µ(mg – F sin θ)
M1
T For the block to move,
T
F1
f2
F cos θ = fL = µ(mg – F sin θ)
M2 a
µmg
or F =  … (i)
F2 cos θ + µ sin θ
θ
For F to be minimum, denominator should be maximum.
Both the blocks will be moving downwards with same d
acceleration a. Different forces acting on two blocks are or (cos θ + µ sin θ) = 0

as shown in figures.
or – sin θ + µ cos θ = 0
P hysi cs | 4.117

or tan θ = µ or θ = tan–1 (µ) Therefore, minimum value of ω should be


Substituting this value of θ in Eq. (i), we get g 9.8
wmin = = rad/s
Fmin = mg sinθ R 0.1
or wmin = 9. 89 rad/s
Sol 7: (a) To find tension at mid-point of the lower g
(b) Eq. (iii) can be written as h = R –
wire we cut the string at this point. Draw the free body ω2
diagram of lower portion. If R and ω are known precisely, then
T1 ∆g
Dh = – 2 or Dg = w2 Dh (neglecting the negative
a ω
sign)
0.1kg
1.9kg (Dg)min = (wmin)2 Dh, (Dg)min = 9. 8 ×10–3 m/s2

(1.9 +0.1)g =2.0 g Sol 9: (a) CP = CO = Radius of circle (R)


∴ ∠CPO = ∠POC = 60°
The equation of motion gives
∴ ∠OCP is also 60°
T1 - 2.0g = (2.0) a
Therefore, DOCP is an equilateral triangle.
or T1=(2.0)(g + a) = (2. 0)(9.8 + 0.2) = 20 N

Sol 8: Given : R = 0 1m, m = 10–2 kg

C
R R–h
N θ
r
h 60° P
mg
O

(a) FBD of particle in ground frame of reference is shown


Hence, OP = R
in figure. Hence,
r
tan θ =
R –h

N cos θ = mg  ... (i)


C
and N sin θ = mrw2  ... (ii) N

Dividing Eq. (ii) by Eq. (i), we obtain F P


2 2
rω r rω O
tan θ = or =
g R –h g mg
g
or w2 =  ... (iii)
R –h Natural length of spring is 3R/4.
This is the desired relation between ω and h. ∴ Extension in the spring
From Eq. (iii) 3 R R
x=R– =
g 4 4
h=R–
ω2 ⇒ Spring force,
Form non-zero value of h
g
R > 2 or ω > g / R
ω
4 . 1 1 8 | Forces and Laws of Motion

m g R  mg
F = kx =     =
 4
 R  4 ω
m1 m2
The free body diagram of the ring will be as shown. r1 × r2
mg r
Here, F = kx =
4
and N = Normal reaction.
(b) Tangential acceleration aT : The ring will move Free body diagram of m1 and m2 with respect to ground
forwards the x-axis just after the release. So net force m1 m2
T+f T
along x-axis
T = m2r2w2  … (i)
Since, m2r2w2 < m1r1w2
y Therefore, m1r1w2 > T
and friction on m1 will be inward (toward centre)
N
60°
f + T = m1r1w2  … (ii)
60°
F from equations (i) and (ii), we get
O 60°
ax f = m1r1w2 – m2r2w2  … (iii)
mg = (m1r1 – m2r2)w 2

Fx = F sin 60° + mg sin60° = (10 × 0.124 – 5×0. 176) (10)2 N = 36 N


m g 3  3 Therefore, frictional force on m1 is 36 N (inwards)
=  
 + mg  
 2 
 4  2   (b) From eq. (iii)

5 3 f = (m1r1 – m2r2) w2
Fx = mg
8 Masses will start slipping when this force is greater than
Therefore, tangential acceleration of the ring, fmax or
Fx 5 3 (m1r1 – m2r2)w2 > fmax > mm1g
aT = ax = = g
m 8 ∴ Minimum value of ω is
Normal reaction N : Net force along y-axis on the ring
just after the release will be zero. µm1g 0.5 × 10 × 9.8
wmin = =
m1r1 – m2r2 10 × 0.124 – 5 × 0.176
Fy = 0
∴ N + F cos 60° = mg cos 60° wmin = 11.67 rad/s

∴ N = mg cos60° – F cos 60° (c) From Eq. (iii), frictional force f = 0


mg mg 1 mg mg r1 m 5 1
= –   = – where m1r1 = m2r2 or = 2 = =
2 4 2
  2 8 r2 m1 10 2
3mg and r = r1 + r2 = 0. 3 m
N=
8 ∴ r1 = 0.1 m and r2 = 0.2 m
i. e. , mass m2 should be placed at 0.2 m and m1 at 0.1 m
Sol 10: Given,
from the centre O.
m1 = 10 kg, m2 = 5 kg, ω = 10 rad/s
r = 0.3 m, r1 = 0.124 m Sol 11: Acceleration of block A

∴ r2 = r – r1 = 0.176 m Maximum friction force that can be obtained at A is

(a) Masses m1 and m2 are at rest with respect to rotating (fmax)A = µA(mg cos 45°)
table. 2 2mg
= (mg/ 2)=
Let f be the friction between mass m1 and table. 3 3
P hysi cs | 4.119

The FBD of the whole system will be as shown in the


figure
m 2m
A B T T
T T
mg sin45° 2mg sin45°
mg 45° 45° 2mg A fB =2mg B
= =F2 = =F1 3
2 2 mg =(fB)max
F2= fA = 2mg
2
mg F1=
Similarly, 3 2 2
(fmax)B = µB(2mg cos 45°)
Therefore, friction on A is
1 2mg
= (2mg/ 2 ) = fA = mg/3 2 (down the plane)
3 3
Therefore, maximum value of friction that can be Now for tension T in the string, we may consider either
obtained on the system is equilibrium of A or B
2 2mg Equilibrium of A gives
(fmax) = (fmax)A + (fmax)B =  ... (i)
3
mg mg 4mg 2 2mg
Net pulling force on the ststem is T = F2 + fA = + = or
2 3 2 3 2 3
2mg mg mg
F = F 1 – F2 = – =  ... (ii)
2 2 2 Similarly, equilibrium of B gives T + fB = F1
2mg 2mg 4mg
From Eqs. (i) and (ii), we can see that or T = F1 – fB = – =
2 3 3 2
Net pulling force < fmax
2 2mg
Therefore, the system will not move or the acceleration or
3
of block A will be zero. 2 2mg
Therefore, tension in the string is
(b) and (c) Tension in the string and friction at A 3
Net pulling force on the system (block A and B) Sol 12: Acceleration of A down the plane,
F = F1 – F2 = mg/ 2 aA = g sin 45° – µA g cos 45°
Therefore, total friction force on the blocks should also  1   1 
mg = (10)   – (0. 2)(10)   = 4 2 m/s
2

be equal to  2  2
2
Similarly acceleration of B down the plane,
or fA + fB = F = mg/ 2
aB = g sin 45° – µB g cos 45°
Now since the blocks will start moving from block B first
(if they move), therefore, fB will reach its limiting value  1   1 
(10)   – (0.3)(10)   = 3.5 2 m/s
2
first and if still some force is needed, it will be provided
 2  2
by fA
The front face of A and B will come in a line when,
Here, (fmax)B < F
sA = sB + 2
Therefore, fB will be in its limiting value and rest will be
provided by fA. 1 2 1 2
or a t = aBt + 2
2mg 2 A 2
Hence fB = (fmax)B =
3 1 1
× 4 2 × t2 = × 3. 5 2 × t2 + 2
mg 2 2
mg 2mg
and fA = F – fB = – =
2 3 3 2 Solving this equation, we get t = 2s
1 2 1
Further, sA = at = × 4 2 × (2)2 = 8 2 m
2 A 2
Hence, both the blocks will come in a line after A has
travelled a distance 8 2 m down the plane.
4 . 1 2 0 | Forces and Laws of Motion

Circular Dynamics (towards centre) and second along tangential. Along


2
radius net force should be equal to mv and along
Sol 13: (A) Tangential force (F1) of the bead will be given R
by the normal reaction (N), while centripetal force (Fc) is
provided by friction (fr). The bead starts sliding when
tangent it should be equal to O
the centripetal force is just equal to the limiting friction.
maT, where aT is the
ω tangential acceleration in θ
the figure. T
mv 2 v
T – Mg cos θ = and
L F1 L
A x
Mg sin θ = MaT θ
Fc mg sinθ
or aT = g sinθ mg cosθ
mg

Ft is inwards ∴ Correct options are (b) and (c).

Therefore, Ft = ma = mαL = N Sol 16: (B, D) A rotating/revolving frame is accelerating


and hence non-inertial. Therefore, correct options are (b)
∴ Limiting value of friction and (d).
(fr)max = mN = mmαL  … (i)
Sol 17: (A, C) –1 1kg
Angular velocity at time t is ω = at 2 ms
∴ Centripetal force at time t will be ⇒
Fc = mLw = mLa t
2 2 2
 … (ii) 1 kg 5 kg

µ
Equating equation (i) and (ii), we get t =  2m2 
α v1' =  m1 – m2  v +   v2
m +m  1
µ  1 2   m1 + m2 
For t > , F > (fr)max i.e. , the bead starts sliding.
α c 1 –5
–2=   v1 + 0 (as v2 = 0)
In the figure Ft is perpendicular to the paper inwards. 1 +5
∴ v1 = 3 ms–1
Sol 14: (A) Since, the block
rises to the same heights in  m2 – m1   2m1 
v0 v 2'
all the four cases, from =   v2 +   v1
conservation of energy,  m2 + m1   m1 + m2 
speed of the block at
 2×1 
highest point will be same N + mg =0+   (3) = 1 ms
–1

in all four cases. Say it is v0.  6 


Equation of motion will be PCM = Pi = (1)(3) = 3 kg-m/s
mv 20 mv 20
N + mg = or N= – mg
R R P5' = (5) (1) = 5 kg-m/s
R (the radius of curvature) in first case is minimum.
2
Therefore, normal reaction N will be maximum in first PCM 9
case. KCM = = = 0. 75 J
2MCM 2×6
Note In the question it should be mentioned that all the
1
four tracks are frictionless. Otherwise, v0 will be different Ktotal = × 1 × (3)2 = 4. 5 J
2
in different tracks.
∴ Correct options are (a) and (c).
Sol 15: (B, C) Motion of pendulum is part of a circular
motion. In circular motion it is better to resolve the Sol 18: (B) (a) CP = CO = Radius of circle (R)
forces in two perpendicular directions. First along radius ∴ ∠CPO = ?POC = 60°
P hysi cs | 4.121

∴ ∠OCP is also 60° Therefore, tangential acceleration of the ring,


Therefore, DOCP is an equilateral triangle. Fx 5 3
aT = ax = = g
m 8
Normal reaction N Net force along y-axis on the ring just
after the release will be zero
Fy = 0
C
∴ N + F cos 60° = mg cos 60°
∴ N = mg cos60° – F cos 60°
60° P
O mg mg  1  mg mg
= –   = –
2 4 2 2 8
Hence, OP = R
3mg
N=
8

Sol 19: (A, C)

C
N

F P
O

Natural length of spring is 3R/4. If θ = 45º then cos θ = sin θ hence block will be at rest.
3 R R
∴ Extension in the spring x = R – = If plane is rough & θ > 45° then sin θ > cos θ so friction
4 4
will act up the plane
m g R  mg
⇒ Spring force, F = kx =    4  = 4 If plane is rough & θ < 45 then cos θ > sin so friction will
 R   
act down the plane so (A, C) are correct
The free body diagram of the ring will be as shown.
mg
Here, F = kx = and N = Normal reaction. Sol 20: (D) Initially bead is applying radially inward
4 normal force.
(b)Tangential acceleration aT the ring will move
During motion at an instant, N = 0, after that N will act
forwards the x-axis just after the release. So, net force
radially outward.
along x-axis
Sol 21: (D) Condition for not sliding,
fmax > (m1 + m2) g sin θ
y mN > (m1 + m2) g sin θ
0.3 m2 g cos θ ≥ 30 sin θ
N
60° 6 ≥ 30 tan θ
60°
F 1/5 ≥ tan θ
O 60°
ax 0.2 ≥ tan θ
mg ∴ for P, Q
m g 3  3 f = (m1 + m2) g sin θ
Fx = F sin 60° + mg sin60° =   + mg  
 4  2  2  For R and S
 
5 3
Fx = mg F = fmax = mm2g sin θ
8

You might also like